Download as pptx, pdf, or txt
Download as pptx, pdf, or txt
You are on page 1of 159

z

Musculoskeletal
Tumors and Diseases
z

CLINICAL SITUATION FOR QUESTIONS 1 THROUGH 4

Figures 1a through 1d are the MR images and biopsy specimen of a


68-year-old man who has a painless mass in his leg. He believes the
mass may have been present for several years, and it is more
apparent now because he recently lost weight after changing his diet
and exercise patterns. He also recently experienced modest trauma
to his leg while moving furniture.
Question 1 of 100
What is the diagnosis z
1 dedifferentiated liposarcoma
2 intramuscular lipoma
3 atypical lipomatous tumor
4 myositis ossificans

Question 2 of 100
The role of surgery in this condition is best described as
1- marginal resection is performed with a low likelihood of recurrence.
2- best performed after the lesion becomes “cold” on a bone scan.
3- wide resection as an indication for curative treatment.
4- not indicated.

Question 3 of 100
The role of radiation treatment for this lesion is
1- proven to decrease local recurrence.
2- associated with a high rate of post-radiation sarcoma development.
3- contra-indicated for benign pathology.
4- associated with a higher risk of wound complications if given post-operatively.

Question 4 of 100
Chemotherapy for this condition is
1- contraindicated when pathology is benign.
2- associated with a high risk for subsequent myelodysplastic syndrome.
3- provides dramatic survival benefits.
4- provides modest survival benefits.
z

This patient has a dedifferentiated liposarcoma within a preexisting atypical lipomatous


tumor. The imaging demonstrates a large fatty mass with increased internal septations
proximally (the atypical lipomatous tumor) and a solid enhancing mass distally (the
dedifferentiated portion). A biopsy reveals a high-grade liposarcoma. The other diagnostic
responses do not reflect sarcomatous transformation of the lesion.

Surgical treatment of a high-grade sarcoma involves wide surgical resection. Radiation


decreases local recurrence but does not clearly influence overall survival. The role of
chemotherapy in high-grade soft-tissue sarcomas remains investigational; there is a
modest (8%-15%) associated improvement in overall survival.

Intramuscular lipomas and atypical lipomatous tumors are treated with marginal resection
alone. Radiation therapy for soft-tissue sarcomas may be given before or after surgery.
When administered before surgery, patients have a higher wound complication rate but
better long-term function attributable to lower rates of lymphedema, fibrosis, and
contractures.
z

Figures 5a and 5b are the radiographs of a 74-year-old man with poorly differentiated squamous cell carcinoma of the lung. He has had an uneventful recovery
after undergoing a wedge resection of his left upper lobe 6 months ago. He is experiencing left lateral knee pain, and a whole-body positron emission
tomography/CT scan shows no avid area other than the lateral left distal femur. This patient has needed to use a wheelchair for 3 weeks because of his pain. You
discuss these treatment options: aggressive curettage, local adjuvant treatment, cementation, and prophylactic fixation vs distal femoral resection and
megaprosthesis total knee arthroplasty reconstruction. You should tell him that

1 his overall disease free surgery will be no different with either procedure
2 fixation failure may occur with cementation and prophylactic fixation but not with megaprosthesis
3 infection rates with megaprosthesis reconstruction are lower than with cementation after aggressive curettage.

4 radiation will not be necessary after undergoing either procedure


z
discussion

Distal femoral megaprosthetic reconstruction after tumor resection is a


reliable oncologic procedure, but 5-year implant survival is as low as 74%
with an approximate 8% deep infection rate. The amputation rate is as high
as 8% because of infection or recurrence, and there is an overall 18%
revision rate. More than 10% of distal femoral megaprosthetic reconstructions
are performed to address metastatic disease.

Fixation failure and infection may occur with either procedure. Radiation may
not be recommended after a megaprosthesis reconstruction unless margins
are not free of tumor. Either operation may be equally successful in returning
patients to functional activities. Overall disease-free survival is related to the
aggressiveness of the tumor and not the type of reconstruction performed.
z
Question 6 of 100
Figures 6a through 6d are the radiographs and T1-weighted sagittal and fat-saturated axial MR images
of an otherwise healthy 56-year-old man who has anterior knee pain and intermittent swelling after
sustaining a noncontact twisting injury. Low-power and high-power hematoxylin and eosin stained
histologic specimens are shown in Figures 6e and 6f. Based on the history, radiographs, CT scan, MR
imaging, and histologic findings, what is the most likely diagnosis?
1 localized pigmented villonodular synovitis (PVNS)
2 synovial hemangioma
3 Synovial chondromatosis
4 Biphasic synovial sarcoma
z
discussion

The lesion in the posterior intercondylar knee notch is a benign synovial hemangioma.
Intralesional calcifications, classically associated with hemangiomas, are frequently not
identified on plain radiographs. The MR imaging reveals a hypervascular lesion with
multiple filling defects, with hyperintensity on T2-weighted images and low-to- intermediate
signal intensity on T1-weighted images. Histologically, vascular lakes within fine capillaries
with a synovium on the surface of the lesion are characteristic of this condition. Many
patients with synovial hemangioma have pain, swelling, stiffness, or mechanical
symptoms. The correlation of symptoms with the hemangioma for this patient is unclear
because there was recent trauma and a concurrent meniscus tear. Simultaneous treatment
of both potential sources of pain is typically recommended. As with PVNS, the disease can
be localized or diffuse. Surgical excision, either open or arthroscopic, is the recommended
treatment. PVNS is the most common intra-articular tumor, but hypointensity in either the
diffuse or localized type is characteristic in both T1- and T2-weighted images. Synovial
sarcoma, although often found close to a joint, is not characteristically found within a joint.
z

Question 7 of 100
Figures 7a through 7d are the radiograph, MR
images, and biopsy specimen of a 35-year-old
man who has a painful, slowly enlarging knee
mass. Which chromosomal translocation is
characteristic of this pathology?
1- t(11;22)
2- t(9;22)
3- t(x;18)
4- t(12;16)
z
discussion

Synovial sarcoma is a soft-tissue sarcoma that usually occurs in young adults.


Synovial sarcoma often causes pain, unlike most soft-tissue sarcomas, which
generally do not cause pain. Imaging characteristics include soft-tissue
calcifications on plain radiographs and a heterogeneous mass that is generally
isointense to muscle on T1-weighted images and hyperintense to muscle on T2-
weighted images. There are biphasic and monophasic types of synovial sarcoma.
The biphasic type, which is depicted here, has both spindle cell and epithelial
components and will stain for both vimentin and cytokeratin. More than 90% of
patients with synovial sarcoma have a characteristic genetic translocation of
t(X;18), which results in the fusion protein SS18-SSX. This translocation can be
stained for use of florescence in situ hybridization technology. t(11;12) is seen in
Ewing sarcoma. T(9;22) is seen in extraskeletal myxoid chondrosarcoma. t(12;16)
is seen in myxoid liposarcoma.
z

Question 8 of 100

A 45-year-old woman has a painless thigh mass that is larger than


5 cm. What is the best next step?

1- Percutaneous biopsy
2- Positron emission tomography (PET)/CT scan 3- Excisional
biopsy
4- MRI of the thigh with gadolinium
z
discussion

Masses exceeding 5 cm in size and any deep mass should be


evaluated with MRI prior to biopsy or excision to ensure the most
viable tissue is sampled and to minimize morbidity and complications
from an improperly placed biopsy site. Examinations are unreliable
when attempting to determine if a mass is a simple lipoma, and any
large or deep mass should be considered a sarcoma until proven
otherwise. PET/CT is a staging examination to evaluate for metastatic
or multifocal disease. These are expensive tests that should not be
ordered prior to MR imaging of the primary lesion. For patients that
are unable to obtain an MRI, CT of the mass is the preferred imaging
modality.
Clinical situation for question 9 through 11Figures 9a through 9d are the anteroposterior and lateral
radiographs, CT scan, and technetium bone scan of a 12- year-old boy who has experienced 7
months of pain in his lower leg. The pain limits his ability to participate in sports and he is having
z difficulty sleeping. He is afebrile, and laboratory study findings including an erythrocyte
sedimentation rate, C-reactive protein, and complete blood count are within normal limits.

Question 9 of 100
What is the most likely diagnosis?
1- Osteomyelitis
2- Osteoid osteoma
3- Stress fracture
4- Adamantinoma

Question 10 of 100
The most appropriate treatment of this lesion involves
1- radiofrequency ablation (RFA).
2- wide resection and hemicortical allograft reconstruction. 3- prophylactic internal fixation followed by radiation.
4- local debridement and an infectious disease consultation.

Question 11 of 100
If this lesion occurred in the spine, which features would most likely be present?
1- Syrinx and paralysis
2- Spondylolisthesis and radiculopathy 3- Epidural abscess and fever
4- Scoliosis and paraspinal pain
z
discussion

The images and clinical history support a diagnosis of osteoid osteoma, which most commonly
occurs in adolescence. Although these lesions can be seen in any bone, they are usually
located in the femur and tibia. The significant inflammatory response to this tumor is secondary
to high levels of prostaglandin production. Characteristic night pain is relieved with nonsteroidal
anti-inflammatory drugs (NSAIDs) or by aspirin.

Radiographic images show thickened bone and a small central nidus. Thin-cut CT scan is the
imaging of choice to visualize the nidus. A bone scan is associated with uptake but is not
specific. Treatment options include expectant management with NSAIDs and observation under
the premise that these lesions eventually burn out. Contemporary treatment involves RFA.
Historically, these lesions were treated with en bloc resection; however, this technique has
largely fallen out of favor because of the high efficacy and comparative low morbidity associated
with RFA.

When an osteoid osteoma occurs in the spine, it is located in the posterior elements, and
paraspinal pain and scoliosis often are present
z

Question 12 of 100

Figures 12a and 12b are a recent radiograph and a whole-body bone scan of an 81-year-old man who has hip pain and
difficulty walking. His medical history is significant for obesity, hypertension, chronic kidney disease, and coronary
artery disease. An examination demonstrates moderate tenderness with passive range of motion of the left hip and an
inability to actively flex the left hip against gravity. What is the best next step?

1- Dynamic hip screw


2- Long cephalomedullary nail
3- Staging studies
4- Toe-touch weight-bearing activity for 6 weeks
z
discussion

This patient has no known history of active malignancy. The


radiograph shows a lesser trochanteric avulsion fracture (a fracture
routinely associated with an underlying neoplasm). The bone scan
reveals no other bone lesions. The femur fracture is statistically
most likely to occur because of metastatic disease, but, without
other evidence of metastasis, a primary bone sarcoma is possible,
and biopsy is recommended before surgical fixation. Observation of
this fracture, which is pathognomonic for neoplastic disease, is
strongly discouraged.
Figures 13a and 13b are the radiographs of a 57-year-old man who is seen
in the emergency department. He has been experiencing left thigh pain for
2 month. Four years ago he underwent laparoscopic nephrectomy and
z states that he has been disease free since the resection (although he has not
seen a doctor in 2 years). The pathogenesis of osteolysis in renal cell
carcinoma metastatic to bone includes secretion of parathyroid hormone-
related peptide (PTHrP), transforming growth factor-B (TGF-B), and
vascular endothelial growth factor (VEGF), which directly cause
overexpression receptor activation of nuclear factor kB ligand (RANKL)
on which cells?

1- Osteoblasts
2- Osteoclasts
3- Osteoclast precursors
4- Both osteoclast precursors and the mature osteoclast
z
discussion

Tumor cells in renal cell carcinoma interact with the bone microenvironment to
drive bone destruction and tumor growth by secreting factors such as PTHrP,
TGF-B, and VEGF. These factors stimulate the host osteoblast, causing
overexpression of RANKL, which in turn causes bone resorption through
stimulation of osteoclasts. RANKL expression is upregulated in many types of
metastatic cancer to bone, and blocking the RANK/RANKL interaction prevents
progression of metastases.

Other actions of RANKL include triggering the migration of human tumor cells
that express RANK. RANK and RANKL are expressed in metastatic renal cell
carcinoma, and their presence strongly signifies potential recurrence. The use
of denosumab, which binds and inactivates RANKL, has its basis in these
findings in renal cell carcinoma.
z

Question 14 of 100

Figures 14a through 14c are the MR images of a 72-year-old man who has had a slow-growing
asymptomatic mass in his thigh for more than 5 years. Cytogenetic testing on the mass reveals a ring
chromosome and MDM2 expression with no 12;16 translocation. What is the most likely diagnosis?

1- Myxoid liposarcoma
2- High-grade pleomorphic liposarcoma 3- Atypical lipomatous tumor
4- Intramuscular lipoma
z
discussion

This slowly growing mass has lipomatous features on MRI with a fat signal
noted on T1 and T2 fat-suppressed images. Some thin striations seen on
MRI may suggest an atypical lipomatous tumor. The molecular changes
confirm that this is not a simple lipoma. High-grade sarcomas are generally
rapidly growing and typically do not have the significant amount of largely
homogenous fat signal as seen on this MRI. The negative result of a 12;16
translocation makes the diagnosis of myxoid liposarcoma unlikely. Atypical
lipomas typically have a ring chromosome and express MDM2 but do not
have the 12;16 translocation, as demonstrated in this patient. Atypical
lipomas are synonymous with well-differentiated liposarcomas and pose
risk for local recurrence but do not pose significant risk for metastatic
spread.
z

Question 15 of 100

Figures 15a through 15c are the radiograph and MR images of a 16-year-old girl who experienced posterior knee pain after a dance recital 3 weeks ago; the pain
resolved 1 week ago with ibuprofen use. What is the most appropriate treatment for this patient?
1- Image-guided core needle biopsy
2- Clinical observation and serial radiographs
3- Tc-99 whole-body bone scan
4- CT scan with sagittal and coronal reconstructions
z
discussion

The images reveal a small reactive-type lesion in the posteromedial aspect


of the distal femur consistent with an avulsive cortical irregularity. Also
referred to as a cortical desmoid, periosteal desmoid, or “tug lesion,” this
lesion is seen most commonly in young adolescents, with a slight
preponderance in boys, with one-third occurring bilaterally. It is thought to
be related to repeated microtrauma from pulling of the adductor magnus or
medial gastrocnemius on their respective periosteal attachment sites.
Proper treatment involves recognition of this benign disorder without further
workup. Often best seen on an oblique radiograph, the lack of soft-tissue
mass or bone destruction leads to the benign diagnosis. Serial radiographs
typically show complete resolution by age 20.
z

Question 16 of 100

Figures 16a through 16c are the radiograph, MR image, and biopsy specimen of a 12-year-old boy who injured his leg during a soccer game. Assuming
other staging study findings are negative, what is the Musculoskeletal Tumor Society (MSTS) stage of this lesion?

1 IA

2 IIA

3 IB

4 IIB
z
discussion

The MSTS staging system assigns progressively higher degrees of


risk to neoplasms based on their surgical grade, anatomic location,
and presence or absence of metastases. Lesions that are low grade
are assigned a score of I, while high-grade lesions are assigned a
score of II. Lesions contained in the bone or those that are
intracompartmental are designated as A, and extracompartmental
lesions are designated as B. Metastases elevates the score to III. the
radiograph and biopsy specimen reveal a high-grade bone sarcoma
with a soft-tissue component with no evidence of metastatic disease.
Therefore, this is a stage IIB lesion.
z
Question 17 of 100

A 57-year-old man has a bone lesion that was identified on radiograph and MR
imaging (Figures 17a and 17b) that were taken to evaluate anterior knee pain. An
examination reveals a positive patellar apprehension test finding. The patient brings
his imaging findings to his appointment, and you learn that an image-guided core
needle biopsy was performed based upon the radiologist’s interpretation of the
imaging. The core needle biopsy pathology interpretation text reads, “a low-grade
cartilage consistent with either enchondroma or low-grade chondrosarcoma. Clinical
and imaging correlation is recommended.” What is the best next step?

1- Fluorescent in situ hybridization analysis of the biopsy sample for the t(9;22)
translocation

2- Repeat core needle biopsy, sampling a different site within the tumor
3- Observation with serial radiographs or MR imaging
4- Intercalary resection and intramedullary fixation
z
discussion

Low-grade cartilage lesions are among the few diagnoses for which proceeding to definitive surgical treatment
without a definitive histologic diagnosis is appropriate. Extended curettage with adjuvants is an acceptable
treatment option for both enchondroma and low-grade chondrosarcoma of the extremities; as such, it is
appropriate to proceed to definitive treatment when these are the only diagnoses in the differential. There is no
consensus regarding the number and type of adjuvants used in intralesional curettage. There is also lack of
consensus regarding how to reconstruct the resultant defect, with some favoring cement and others favoring
bone grafting. A wide resection with intercalary resection is not indicated. Most commonly, serial radiologic
studies are recommended to ensure no change occurs over time when the lesion is found incidentally, as in this
patient.

The t(9;22) translocation occurs in extraskeletal myxoid chondrosarcoma, which is not a consideration for this
patient based upon location, imaging, and the biopsy interpretation. Repeat core needle biopsy is not indicated
because the value of core needle biopsy in general for low-grade cartilage lesions is questionable in light of
sampling error concerns and the inability of expert pathologists to reliably distinguish between enchondroma and
low-grade chondrosarcoma. Many orthopaedic oncologists favor proceeding directly to curettage with adjuvants
without any biopsy when the imaging is classic for a low-grade cartilage tumor without any evidence of
dedifferentiation. Enchondromas are frequently positron emission tomography (PET) avid, and PET is not used
to distinguish enchondroma from low-grade chondrosarcoma.
z

CLINICAL SITUATION FOR QUESTIONS 18 THROUGH 21


Figures 18a through 18c are the radiographs and bone scan of a 23-year-old woman. Question 18 of 100
What is the most likely diagnosis?
1- Fibrous dysplasia

2- Metastatic carcinoma
3- Multiple hereditary exostosis
4- Multiple myeloma
Question 19 of 100
z
Despite adequate medical management, the patient continues to experience leg pain that interferes with even the lowest demands of daily living. You
recommend prophylactic intramedullary nailing of the tibia with interlocking screws. Prior to the surgery, you should recommend

1- an echocardiogram.
2- an endocrinology consultation.

3- a serum calcium level.


4- a repeat nuclear bone scan.

Question 20 of 100

The most common extraskeletal manifestation of this disease is

1- café au lait macules.


2- urinary protein elevation. 3- a primary lung mass.
4- an arrhythmia.

Question 21 of 100

The underlying cause of the neoplasm is

1- ALK gene rearrangement.


2- a nongerm-cell mutation of the GNAS1 gene.
3- a germline alteration in EXT1 and EXT2.
4- an abnormality arising from the translocation t(11;14)(q13;q32).
discussion
z

The bone scan reveals multiple bone lesions, which does not rule out any of the responses. The radiographs reveal dysplastic
bone with a “ground glass” appearance, suggesting fibrous dysplasia as the preferred response. Multiple myeloma typically
demonstrates purely lytic, “punched out” lesions and would be highly unusual in a 23-year-old woman. Multiple hereditary
exostosis would demonstrate more expansile lesions concentrated in the metaphysis. Metastatic carcinoma could have a lytic or
blastic appearance but is less likely to occur in a 23-year-old woman. McCune-Albright syndrome in polyostotic fibrous
dysplasia is present in as many as 50% of patients and should be evaluated for during an endocrine consultation. Adrenal,
pituitary, parathyroid, and thyroid endocrinopathies may be present. Untreated hyperthyroidism can be life threatening during a
surgical procedure. There is no indication to repeat the nuclear bone scan. Although phosphate wasting and, rarely, oncogenic
osteomalacia have been reported in polyostotic fibrous dysplasia, an endocrinology consultation always should be sought.

Café au lait macules are the most common extraskeletal manifestation of fibrous dysplasia, often referred to as
“coast of Maine” in appearance because of their irregular borders (in comparison to the “Coast of California”
with smooth borders as seen in neurofibromatosis). Multiple myeloma would not ordinarily appear with
increased uptake on a bone scan unless a pathologic fracture of some duration were present. A long area of bone
involvement would not appear in patients with metastatic lung adenocarcinoma. An ALK rearrangement occurs
in nonsmall-cell lung cancer. The translocation t(11;14)(q13;q32) should be recognized as a poor prognosticator
in multiple myeloma. The germline alteration in EXT1 and EXT2 occurs in multiple hereditary exostosis. All
forms of fibrous dysplasia are caused by a nongerm-cell mutation that occurs during early embryogenesis. A
missense mutation of the GNAS1 gene, which encodes the alpha subunit of the stimulatory G-protein-couple-
z

Question 22 of 100
Figures 22a and 22b are the anteroposterior knee radiograph and an axial T2-weighted MR image of an 11-year-old boy who experienced knee pain following soccer
practice. What is the best approach for biopsy?
1- Lateral parapatellar

2- Medial parapatellar 3- Directly medial


4- Directly anterior
z
discussion

The biopsy should cross only 1 compartment if possible and


proceed as directly as possible to the tumor. The lateral
parapatellar and medial parapatellar approaches cross the knee
joint, potentially visualizing tumor into the knee. A direct anterior
approach will contaminate an extensive portion of the quadriceps
muscle and potentially complicate limb salvage. Biopsy should be
performed at the institution at which definitive resection is planned.
z

Question 23 of 100

Figures 23a through 23c are the MR images of a 55-year-old woman who has experienced more than 10 years of right lower
extremity pain that is radiating along the sciatic nerve distribution even though she has had multiple spine decompression procedures.
She cannot sit comfortably in a chair and feels a fullness in the posterior aspect of her thigh. What is the most likely diagnosis?

1- Malignant peripheral nerve sheath tumor

2- Schwannoma
3- Myxoid liposarcoma
4- Desmoid fibromatosis
z
discussion

This patient has had long-standing radiating pain along the sciatic nerve distribution that
has not been relieved with spine decompression procedures. Localized symptoms to her
posterior thigh should elicit further evaluation with MR imaging. The chronicity of pain
and homogeneous appearance of the lesion on MRI would speak against a malignant
peripheral nerve sheath tumor. MR imaging reveals a large fusiform mass along the
course of the sciatic nerve with a bright T2 signal that is consistent with a nerve sheath
tumor. Neurofibroma is another viable option but is not listed as a response, and the
patient does not have neurofibromatosis. Myxoid liposarcoma is an intermediate-grade
malignancy and could cause long-standing symptoms, but this diagnosis is inconsistent
with the imaging provided. Desmoid fibromatosis has features inconsistent with the
imaging provided; it typically would involve a darker T2 signal than a myxoid liposarcoma
or Schwannoma, and, while sometimes involving a major nerve, this condition typically
does not grow along the course of the nerve like a benign nerve sheath tumor.
z
Question 24 of 100

Figures 24a and 24b are the right femur radiograph and bone scan of a 71-year-old man
with long-standing metastatic prostate cancer who has experienced increasing right thigh
pain for 2 months. The pain is worse with activity and is alleviated with rest. He
experienced similar pain in his left thigh 18 months ago and subsequently sustained a left
subtrochanteric femur fracture after a low-energy twisting injury. He was successfully
treated with an intramedullary nail. He had been receiving zoledronic acid for 4 years
prior to the fracture. This patient’s history includes heavy steroid use. His current
symptoms are most likely the result of

1- a prostaglandin-secreting prostate metastasis.


2- inhibition of osteoclastic function.
3- right L4 radiculopathy secondary to prostate metastasis. 4- direct inhibition of
osteoclast prenylation.
z
discussion

This patient’s history and imaging are most consistent with an atypical
femoral fracture attributable to impaired bone remodeling. Both
bisphosphonates and denosumab are associated with atypical femur
fractures. Atypical fractures may occur in the diaphysis and the
subtrochanteric region. The patient’s prior femur fracture was attributable
to bisphosphonate (zoledronic acid) use. Bisphosphonates directly inhibit
osteoclasts. He subsequently experienced a long symptom-free interval
during which he took denosumab, which is an indirect osteoclast inhibitor.
The imaging is not consistent with a symptomatic prostate metastasis. L4
radiculopathy may cause thigh pain, but there is no evidence (other than
the mention of spine metastasis) supporting this diagnosis.
z

Question 25 of 100

Denosumab, the human monoclonal antibody that specifically binds


and inactivates receptor activator of nuclear factor-kB ligand
(RANKL), is commonly used in the setting of metastatic disease. Its
cell surface receptor is expressed by

1- mature osteoclasts only.


2- mature osteoblasts only.
3- both osteoclast precursors and mature osteoclasts.

4- both osteoblast precursors and mature osteoblasts.


z
discussion

Bone remodeling involves both osteoblasts and osteoclasts. RANKL, a member of the tumor
necrosis factor (TNF) family, is a key regulator of osteoclast formation and function. The
receptor of RANKL, RANK, is expressed by both osteoclast precursors and mature osteoclasts.
Denosumab inhibits the binding of RANKL to RANK, decreasing osteoclastogenesis and bone-
resorption by mature osteoclasts.

Osteoblasts produce osteoprotegerin, which stimulates osteoclasts to induce apoptosis by


acting as a decoy for RANKL, preventing the binding of RANKL to RANK. Osteoprotegerin also
binds to other TNF-related apoptosis- inducing ligands and other TNF members including TNF-
a, TNF-B, and CD40 ligand. Denosumab does not bind to any other TNF members.

Breast cancer metastases produce cytokines that induce osteoblasts to express elevated levels
of RANKL, stimulating osteoclastogenesis via binding to RANK and activating downstream
signaling pathways to osteoclast precursors, causing further bone resorption. Measurements of
bone product turnover (including N-telopeptide) decrease during treatment with denosumab.
z

RESPONSES FOR QUESTIONS 26 THROUGH 31


1- Ultrasound
2- MRI with and without contrast
3- Chest CT scan and whole-body bone scan
4- Positron emission tomography (PET)
5- Presurgical radiation therapy
6- marginal resection
7- Transverse incisional biopsy centered over the mass
8- Incisional biopsy centered over the mass in line with the long axis of the limb
9- Sentinel node biopsy
10- Core needle biopsy

For each clinical scenario or question below, select the most appropriate evaluation or
treatment step listed above.
Question 26 of 100
Figure 26 is the posteroranterior chest radiograph of a 76-year-old man with an atraumatic gradually enlarging mass overlying his left clavicle that
has been present for 6 months. There are no changes in overlying skin. His only noteworthy medical history involves facial squamous cell carcinomas
z
that have been successfully removed surgically.
1- Ultrasound
2- MRI with and without contrast
3- Chest CT scan and whole-body bone scan
4- Positron emission tomography (PET)
5- Presurgical radiation therapy
6- Marginal resection
7- Transverse incisional biopsy centered over the mass
8- Incisional biopsy centered over the mass in line with the long axis of the limb 9- Sentinel node biopsy
10- Core needle biopsy

Question 27 of 100
A 63-year-old man with right hip pain was followed 8 years ago for an incidental intraosseous lesion in the right periacetabular and ischial region that
was isointense with fat on all images. He was discharged from follow-up after 3 years when no change was documented. He began experiencing pain
in his hip, and a bone scan showed grade 3 uptake. New MR imaging was obtained, and an axial image at the level of the hip is shown in Figure 27. A
PET/CT scan shows dramatic activity in the lesion without any other area of activity.
1- Ultrasound
2- MRI with and without contrast
3- Chest CT scan and whole-body bone scan
4- Positron emission tomography (PET)
5- Presurgical radiation therapy
6- Marginal resection
7- Transverse incisional biopsy centered over the mass
8- Incisional biopsy centered over the mass in line with the long axis of the limb 9- Sentinel node biopsy
10- Core needle biopsy
Question 28 0f 100 z
Figure 28 is the MR image of a 65-year-old man with an American Joint Committee on Cancer III anterior arm pleomorphic intermediate- to high-grade
sarcoma. The patient is now considering treatment options. He underwent a wide excision at an outside hospital 2 years previously. The treating surgeon
recommended an amputation, and the patient is now seeking a second opinion. Imaging studies reveal no other sites of disease.
1- Ultrasound
2- MRI with and without contrast
3- Chest CT scan and whole-body bone scan

4- Positron emission tomography (PET)


5- Presurgical radiation therapy
6- Marginal resection
7- Transverse incisional biopsy centered over the mass
8- Incisional biopsy centered over the mass in line with the long axis of the limb 9- Sentinel node biopsy
10- Core needle biopsy

Question 29 of 100
Figure 29 is the MR image of a 44-year-old woman who elects surgical treatment of an intramuscular lipoma in her
dominant right arm.
1- Ultrasound
2- MRI with and without contrast
3- Chest CT scan and whole-body bone scan
4- Positron emission tomography (PET)
5- Presurgical radiation therapy
6- Marginal resection
7- Transverse incisional biopsy centered over the mass
8- Incisional biopsy centered over the mass in line with the long axis of the limb 9- Sentinel node biopsy
10- Core needle biopsy
Question 30 of 100

z
Figure 30 is the MR image of a 29-year-old man with a large and enlarging thigh mass. Needle biopsy findings are inconclusive.

1- Ultrasound
2- MRI with and without contrast
3- Chest CT scan and whole-body bone scan
4- Positron emission tomography (PET)
5- Presurgical radiation therapy
6- Marginal resection
7- Transverse incisional biopsy centered over the mass
8- Incisional biopsy centered over the mass in line with the long axis of the limb

9- Sentinel node biopsy

10- Core needle biopsy

Question 31 of 100

Figure 31 is the sagittal MR image of a 30-year-old man with a clear-cell sarcoma of the foot. There is no evidence of disease elsewhere after standard staging of a soft-tissue
sarcoma.

1- Ultrasound
2- MRI with and without contrast
3- Chest CT scan and whole-body bone scan
4- Positron emission tomography (PET)
5- Presurgical radiation therapy
6- Marginal resection
7- Transverse incisional biopsy centered over the mass
8- Incisional biopsy centered over the mass in line with the long axis of the limb

9- Sentinel node biopsy


10- Core needle biopsy
discussion

For patients with rapidly enlarging painless masses, particularly those that are either large or deep, the diagnosis of a soft-tissue sarcoma should be
entertained. Masses exceeding 5 cm in largest dimension that are subfascial and heterogenous on MRI are concerning. MRI with and without contrast is
the preferred imaging modality for evaluation of soft-tissue sarcomas because it can delineate location of the lesion, involvement of neurovascular
structures, intra-articular involvement, and underlying signal alteration in the osseous structures. Most patients can relate the onset of their symptoms to a
traumatic event, and the interpretation of an MRI may include a hematoma. Ultrasound is more commonly used to confirm the clinical impression of a
cyst and to distinguish cystic from solid masses.

A core needle biopsy can easily be performed in an outpatient setting or with image guidance to aid in the diagnosis. Core needle biopsies preserve the
architectural relationship of cells, which is important in the diagnosis of mesenchymal lesions. A core needle biopsy is appropriate for soft-tissue lesions
and osseous lesions and should be performed in a multidisciplinary setting with a surgeon performing the resection so unnecessary compartmental
contamination is not introduced into the needle tract.

A fine-needle aspiration allows cytologic but not histologic analysis and generally is not favored for the diagnosis of a sarcoma. If an open biopsy is
performed, the incision must be oriented in line with the long axis of the extremity to minimize contamination of surrounding structures. Meticulous
hemostasis and closure of the tumor pseudocapsule must be achieved to minimize local tissue contamination. An open biopsy is commonly used if an
initial needle biopsy is nondiagnostic. Transverse biopsy incisions should not be used.

Presurgical radiation for sarcomas is advantageous to decrease the field of radiation and overall radiation dose, but a significant postsurgical wound-
healing complication rate (up to 35%) is associated with presurgical radiation therapy.

Staging of sarcomas is important to help predict prognosis. Not all sarcomas are reliably positive on a PET scan, so the preferred staging studies are CT
scan of the chest and whole-body bone scan.

Hematougenous spread of sarcomas is the most common route of metastatic disease, which speaks to the value of chest CT scans. Clear-cell sarcomas (in
addition to synovial sarcoma, angiosarcoma, epithelioid sarcoma, and rhabdomyosarcoma) tend to involve lymphatic nodal metastatic disease, so sentinel
node biopsy is considered when assessing these tumors. Evidence supports efficacy of sentinel node biopsy for clear-cell sarcomas in particular.
z

Question 32 of 100
Figures 32a through 32d are the radiographs and MR images of a 13-year-old girl with new posterolateral knee pain following a fall. What is the
best next step?

1- Percutaneous biopsy
2- Open curettage and grafting
3- Referral to a musculoskeletal oncologist
4- Observation with follow-up radiographs
z

discussion

This patient has an incidentally noted benign-appearing fibro-osseous lesion of her proximal tibia. The lesion does
not correspond to her area of pain, and there is no indication she experienced pain from the lesion prior to the fall.
Radiographs and MR imaging findings are consistent with a fibrous cortical defect or nonossifying fibroma. There
is no surrounding edema in the bone or soft tissue and no soft-tissue mass, and the lesion has a sclerotic rim around
it. These are common lesions in children, and there is no need for a biopsy, curettage, or referral to a tumor
specialist. This patient should not be at high risk for pathologic fracture from this lesion. Observation with follow-
up radiographs in 3 to 6 months is the treatment of choice.
z

Question 33 of 100

Figures 33a through 33d are the radiograph, MR images, and biopsy specimen of
a 66-year-old woman with a several-month history of an enlarging thigh mass
after “pulling a muscle” while playing softball. Several weeks ago her physician
aspirated the cyst, but the result is no change. Treatment of this lesion should
include

1- embolization.
2- reaspiration and observation.

3- radiation therapy alone.


4- radiation therapy and surgery.
z discussion

This patient’s imaging and biopsy specimen are consistent with a high-grade undifferentiated
pleomorphic sarcoma (UPS). Histologically, UPS is characterized by high cellularity, marked nuclear
pleomorphism, abundant mitotic activity (including atypical mitoses and a spindle cell morphology.
Necrosis is common and characteristic of high- grade lesions. Treatment consists of wide surgical
excision and, in almost all cases, radiation. Radiation eliminates the need for limb amputation, and there
is level I evidence to show that it leads to equivalent rates of survival. Radiation may be delivered either
before or after surgery depending on the surgeon’s and multidisciplinary tumor board’s recommendations.
There may be a role for presurgical embolization in some cases of high-grade soft-tissue sarcoma, but this
is not appropriate as a standalone treatment. Chemotherapy provides limited survival improvement in
adults with high-grade soft-tissue sarcomas.
z

Question 34 of 100

A 79-year-old woman is seen for follow-up of a right arm posterior compartment high-
grade soft-tissue sarcoma after undergoing wide resection of the tumor with preservation
of the radial nerve and minimal stripping of the posterior humeral periosteum. She then
received 70 Gy of postsurgical radiation. Local recurrence occurred 4 years later, and
she was treated with re-resection and adjuvant doxorubicin and ifosfamide
chemotherapy. At that time, she learned she had osteoporosis and was treated with
alendronate. She experienced an atraumatic fracture 2 years later without evidence of
local recurrence. Which treatment poses highest risk for fracture in this scenario?

1- Chronic oral bisphosphonate use


2- Doxorubicin and ifosfamide chemotherapy

3- Timing and dose of radiation


4- Surgical periosteal stripping
z
discussion

Although postsurgical radiation is associated with lower rates of wound


complication, radiation that is administered after surgery necessitates higher
radiation doses and a larger tissue field than radiation administered before surgery.
Risk for radiation-associated fracture increases with larger fields and higher doses.
Retrospective studies have demonstrated increased fracture risk with postsurgical
radiation doses of at least 60 Gy compared with presurgical radiation doses of 50
Gy. The influence of periosteal stripping on fracture risk is controversial. In a series
of patients receiving primarily postsurgical radiation, extensive stripping was an
accepted risk factor, but it was not found to be a significant risk factor in a more
recent analysis of patients receiving lower-dose presurgical radiation. Cytotoxic
chemotherapy is detrimental to healing of established fractures but does not
correlate with risk for fracture in soft-tissue sarcoma. Chronic bisphosphonate use
is associated with atypical fractures, but the location, timing, and radiographic
appearance in this scenario are not consistent with this etiology.
z
CLINICAL SITUATION FOR QUESTIONS 35 THROUGH 37
Figure 35 is the intraoperative finding of a 28-year-old man with a recurrent nontraumatic effusion who undergoes
arthroscopy.
Question 35 of 100
A neoplasm that involves rearrangements of 1p13 involving the colony-stimulating factor 1 (CSF1) gene which, when
expressed, causes proliferation of neoplastic cells and the recruitment of monocyte-macrophage non- neoplastic cells
is
1- a tenosynovial giant-cell tumor.
2- a nodular fasciitis.
3- an infantile fibrosarcoma.
4- an inflammatory myofibroblastic tumor.
Question 36 of 100
This tumor has been recently treated in phase 1 trials with molecularly targeted therapies including a conformation-
specific inhibitor of CSF1 receptor (CSF1R), resulting in at least a 50% reduction of tumor volume in some patients.
This type of inhibitor is further defined as
1- a tyrosine kinase inhibitor.
2- a substrate binding synthetic molecule.
3- an anti-CSF1R antibody.
4- an anti-CSF1 antibody.
Question 37 of 100
What is the neoplastic cell of origin for this tumor?
1- Lymphocyte
2- Monocyte
3- Macrophage
4- mononuclear phagocyte
discussion
z

Tenosynovial giant-cell tumors are widely known as pigmented villonodular synovitis (PVNS), although this term is
misleading because this tumor type is a clonal neoplasm and does not involve an inflammatory process. It often is shown to
have a t(1:2)(p13q37) karyotype resulting in CSF1-COL6A3 gene fusion. There are various amounts of mononuclear cells,
osteoclastlike giant cells, foamy histiocytes, hemosiderophages, and chronic inflammatory cells. Local recurrences are
common, but CSF1R inhibitors are being investigated in studies involving local control improvement and disease
regression.
Targeted therapy trials to assist in control of the diffuse-type tenosynovial giant-cell tumor (formerly called PVNS) involve the use of
monoclonal antibodies that inhibit CSF1R activation. CSF1R-expressing mononuclear phagocytes are affected by these monoclonal
antibodies.
Infantile fibrosarcoma is associated with the t(12;15)(p13;q25) karyotype and ETV6-NTRK3 gene fusion product. Nodular fasciitis is
associated with the t(17;22)(p13;q13.1) karyotype and MYH9-USP6 gene fusion product. Inflammatory myofibroblastic tumor is
associated with translocations involving 2p23 resulting in multiple fusion products of ALK with TPM4 (19p13.1), TPM3 (1q21),
CLTC (17q23), RANBP2 (2q13), ATIC (2q35), SEC31A (4q21), and CARS (11p15). No nonpreferred response has a histologic
appearance that includes hemosiderin, foamy histiocytes, and osteoclastlike giant cells.
A conformation-specific inhibitor of the juxtamembrane region of CSF1R is a synthetic molecule that is designed to access the
autoinhibited state of the receptor through direct interactions with the juxtamembrane residues embedded in the adenosine 5’-
triphosphate-binding pocket. It is designed to bind in the regulatory a-helix of the N-
terminal lobe of the kinase domain in neoplastic cells of tenosynovial giant-cell tumor that have expression of the CSF1 gene. There is
a structural plasticity of the domain of the CSF1R that allows the molecule to directly bind the autoinhibited state of CSF1R.
Another approach involves the development of the anti-CSF1R antibody, emactuzumab, which targets tumor- associated macrophages.
A lower percentage of volume reduction has been reported with imatinib, a tyrosine kinase inhibitor. Alkylating agents have not been
used in this benign neoplasm.
Tenosynovial giant-cell tumor is characterized by an overexpression of CSF1. CSF1R activation leads to recruitment of CSF1R-
expressing cells of the mononuclear phagocyte lineage.
z

Question 38 of 100

Figures 38a and 38b are the radiographs of a 12-year-old girl with a slowly enlarging mass on her
posterior elbow. She has an unremarkable medical history. The mass is nontender, soft, and mobile
with respect to the underlying bone. Her elbow motion is supple and unrestricted. What is the most
likely cause of this condition?

1- Inherited abnormality of phosphate metabolism

2- Inherited abnormality of calcium metabolism

3- Renal disease with chronic renal insufficiency

4- Chronic hyperparathyroidism
z
discussion

The history, examination, and radiographs strongly suggest tumoral


calcinosis. Inherited forms of tumoral calcinosis are often
attributable to a genetic disorder involving phosphate homeostasis.
Acquired tumoral calcinosis is usually secondary to chronic renal
disease and dialysis with secondary hyperparathyroidism.
z

Question 39 of 100

Figures 39a through 39c are the radiographs and MR image of a 14-year-old boy who has intermittent
knee swelling and pain exacerbated by activity. What is the most likely diagnosis?

1- Vascular malformation
2- Synovial chondromatosis
3- Diffuse pigmented villonodular synovitis (PVNS)

4- Lipoma arborescens
z
discussion

This patient’s history does not rule out any of these diagnoses. Imaging
shows multifocal areas of ropey vascular malformations with signal voids in
the thigh. The MR image shows intra-articular involvement that is often the
most symptomatic area in these patients. The radiographs do not show the
classic phleboliths seen with venous malformations or the typical
calcifications of synovial chondromatosis. Lipoma arborescens would involve
an overabundance of fat throughout the joint noted on MRI, and PVNS will
typically involve an effusion, diffuse synovial hyperplasia, and multiple focal
nodules of disease. The appropriate treatment for intra-articular vascular
malformations has not been established. Compression garments should be
considered initially, with persistent symptoms warranting arthroscopic or open
excision of the vascular malformations with synovectomy.
z

Question 40 of 100

A 21-year-old man learned he had Ewing sarcoma of the right distal femur at age 13. He
underwent treatment with neoadjuvant chemotherapy, surgical resection with distal femoral
replacement, and adjuvant chemotherapy. He completed treatment at age 14, and
subsequent surveillance imaging has shown no evidence of recurrence. He has noted
increasing fatigue during the last 3 months, however, and bleeding when he brushes his
teeth. An examination is unremarkable with the exception of bilateral pretibial bruising for
which the patient does not recall trauma. Which tests will aid in diagnosis?

1- Erythrocyte sedimentation rate (ESR) and C-reactive protein (CRP)

2- Serum protein electrophoresis


3- Complete blood count (CBC) with a peripheral smear
4- Platelet aggregation testing and bleeding time
z
discussion

This patient most likely has therapy-associated leukemia, which occurs in approximately 2% of
patients who have Ewing sarcoma within 5 years of receiving treatment. Most cases are
myelodysplasia or acute myeloid leukemia. Most patients with therapy-associated leukemia
respond poorly to conventional chemotherapies and require allogenic stem cell transplant for
cure. Fatigue, gum bleeding, and bruising should raise concern for leukemia in light of his
clinical history. A CBC with a peripheral smear will likely reveal an elevated leukocyte count with
an abnormal percentage of myeloblasts. ESR and CRP are required to assess for infection of a
prosthesis. Fatigue may be associated with chronic infection, but bleeding would be atypical.
Platelet aggregation test results may be abnormal for this patient, but this testing is unlikely to
reveal a specific diagnosis because drugs and genetic disorders may cause platelet
aggregation dysfunction. Serum protein electrophoresis is diagnostic for multiple myeloma, but
this patient is the wrong age for this condition and does not have appropriate signs and
symptoms. A CBC can demonstrate early findings for leukemia with anemia and platelet
suppression despite no blast presence. Of note, 25% of long-term survivors of Ewing sarcoma
have findings of cardiotoxicity.
z

Question 41 of 100
Figures 41a through 41d are the radiograph, MR images, and biopsy specimen of a 35-year-old woman
with pain and progressive paresthesias in her left arm. Staging shows no other lesions. Appropriate local
control for this condition requires

1- forequarter amputation.
2- Tikhoff-Linberg limb salvage resection.
3- contaminated wide resection with a postsurgical radiotherapy boost.
4- chemotherapy and radiotherapy alone.
z
discussion

This patient has a pathologic fracture from a high-grade chondrosarcoma with


encasement of the axillary contents. Meaningful wide resection necessitates amputation.
Radiotherapy and chemotherapy do not have a defined role in the treatment of
conventional chondrosarcoma. The resection of the axillary, musculocutaneous, or radial
nerve or all 3 nerves is not an indication for a forequarter amputation. If the median or
the ulnar nerve is expected to be resected, forequarter amputation should be seriously
considered. An absolute indication of a forequarter amputation is encasement of the
vascular bundle. Another strong indication is a pathologic fracture.

In 1 study that included both chondrosarcoma and dedifferentiated chondrosarcoma with


pathologic fractures involving the proximal femur, the overall 5-year survival rates were
57% and 0%, respectively. In another study, local recurrence was a prelude to distant
metastasis, and tumor grade and size and adequacy of the resection trended toward
predictors of outcome.
z

Question 42 of 100

Figures 42a through 42d are the radiograph, CT scans, and biopsy specimen of a 38-year-old
man who arrived at the emergency department with urinary retention. He has no other
symptoms. What is the most appropriate treatment for this lesion?

1- Surgery alone
2- Surgery and radiation therapy
3- Surgery, radiation therapy, and chemotherapy

4- Radiation therapy alone


z
discussion

These images reveal a chondrosarcoma. On radiograph,


chondrosarcoma is a fusiform, lucent defect with scalloping of the
inner cortex and periosteal reaction. Extension into the soft tissue
may be present, as well as punctate or stippled calcification of the
cartilage matrix. Histologically, chondrosarcoma is differentiated
from benign cartilage tumors by enlarged plump nuclei, multiple
cells per lacunae, binucleated cells, and hyperchromic nuclear
pleomorphism. Treatment of most chondrosarcomas is surgery only
because adjuvant treatments are not effective.
z

Question 43 of 100

Which treatment regimen for Ewing sarcoma most effectively


controls disease?

1- Chemotherapy alone
2- Chemotherapy plus wide surgical resection

3- Chemotherapy plus radiation therapy


4- Surgery plus radiation therapy
z
discussion

Patients with Ewing sarcoma need chemotherapy to treat


micrometastatic disease. Historical outcomes of patients who receive
local control alone without chemotherapy are dismal. Chemotherapy
alone, however, is not adequate to control local bulky disease. Local
control options include radiation therapy or wide surgical resection.
Historically, it was believed that surgery should be recommended for
expendable bones to minimize morbidity. More recent data support
improved outcomes (vs outcomes associated with radiation alone to
the primary site of disease) for patients who undergo wide surgical
resection; consequently, chemotherapy plus wide surgical resection is
the most effective regimen.
z

Question 44 of 100
Figures 44a through 44c are the clinical photograph and radiographs of a 59-year-old man who has a 4-year history of metastatic renal cell carcinoma to the brain, lungs, and
bones.
He has been referred for a painful left proximal femur metastasis. Axial MR images are shown in Figures 44d and 44e. Presurgical embolization, en bloc resection, and
proximal femur replacement are performed. The resection specimen and a postsurgical radiograph are shown in Figures 44f and 44g. Compared with intramedullary nail
fixation, this treatment strategy
1- entails a higher perioperative mortality risk.
2- improves overall survival.
3- poses lower risk for local treatment failure.
4- is associated with less complete pain relief.
z
discussion

Both a retrospective study and retrospective review of the prospectively collected Scandinavian Sarcoma Group database have shown
lower rates of revision surgery and local control failure with en bloc resection than with intralesional procedures. Multiple retrospective
studies of solitary renal cell metastases have demonstrated an overall survival benefit with en bloc resection; however, these data are not
applicable because this patient has multisystem disease. A perioperative mortality difference between resection and nailing procedures has
not been demonstrated. Although surgical intervention to treat pathologic fractures improves pain, studies comparing the adequacy of
different interventions are limited. In this patient, a nail procedure would necessitate less surgical dissection and shorter surgical time.
However, a nail would not address the large soft-tissue mass that likely was a major contributor to this patient’s pain.
Question 45 of 100
z
Figures 45a and 45b are the radiographs of a previously
asymptomatic 10-year-old girl who fell off of her bike. Treatment of
this lesion should consist of

1- observation.
2- MRI with and without contrast.

3- curettage and bone grafting.


4- open biopsy.
z
discussion

Nonossifying fibroma of bone is a common entity and consists of a


solitary eccentric, lytic expanded lesion in the metaphysis of a long
bone. Clinically, nonossifying fibromas are asymptomatic and are
usually an incidental finding on radiograph. These lesions normally
regress spontaneously.
z

Question 46 of 100

Figure 46a is the lateral radiograph of a 54-year-old man who has had a painless soft-tissue mass on his right foot
that has been growing slowly for about 1 year. MR sequences are shown in Figures 46b through 46e. A biopsy is
performed, and a low-power hematoxylin and eosin photomicrograph is seen in Figure 46f. The most appropriate
treatment for this lesion is

1- a diet that reduces uric acid production.


2- wide local resection followed by radiotherapy.
3- marginal excision.
4- excision and radiotherapy after the mineralization matures
z
discussion

This is a giant-cell tumor of the tendon sheath. A solid lesion larger than 1.5 cm in
the hand or foot warrants biopsy to rule out malignancy. The 3 most common
benign tumors of the foot are ganglion cysts, giant-cell tumors of tendon sheath,
and hemangiomas. Radiographs are nonspecific but may reveal extrinsic erosions
of bone by a giant-cell tumor of the tendon sheath. MR findings of prominent
low-signal intensity (seen with T2 weighting) and “blooming” artifact from the
hemosiderin (seen with gradient-echo sequences) are nearly pathognomonic of
this diagnosis, as seen here. Pathology shows hemosiderin-laden macrophages
and giant cells as seen in the histology. Uric acid reduction would be indicated for
gout and wide local excision for a soft-tissue sarcoma. Observation awaiting
mature mineralization would apply to heterotopic ossification.
z

Question 47 of 100
Figures 47a and 47b are the radiograph and axial CT section of a 73-year-old woman with metastatic lung
cancer
who has a painful left periacetabular lesion. She is a high-risk surgical candidate because of a prior
pneumonectomy and progressive metastatic disease of her remaining lung. Palliative radiation is
recommended. Two regimens are being considered: a single fraction of 8 Gy or 15 fractions of a 30-Gy
cumulative dose over 3 weeks. Compared to the multifraction regimen, the 8-Gy single fraction is
associated with
1- inferior pain relief.
2- higher cost.
3- superior pain relief.
4- equivalent pain relief.
z
discussion

Multiple prospective randomized controlled trials have evaluated single vs


multifraction radiation regimens for the treatment of painful bone metastases. All
studies have demonstrated equivalent pain relief. Retreatment rates are higher
with single-fraction dosing, but it is unknown if this is because of higher rates of
recurrent pain with single fractions or reluctance of radiation oncologists to give
additional radiation when multifraction regimens with higher cumulative doses
have failed. Single-fraction radiation is less expensive and more convenient for
patients. The American Board of Internal Medicine Foundation’s Choosing
Wisely® campaign to encourage physician leadership in reducing harmful or
inappropriate resource use selected “Don’t recommend more than a single fraction
of palliative radiation for an uncomplicated painful bone metastasis” as 1 of the “5
things physicians and patients should question in hospice and palliative medicine.”
z

Question 48 of 100

Figures 48a through 48e are the MR image, radiograph, CT scan, and biopsy
specimens of a 14-year-old boy with right shoulder pain without antecedent
trauma. What is the most likely diagnosis?

1- Unicameral bone cyst

2- Eosinophilic granuloma

3- Chondrosarcoma
4- Chondroblastoma
z
discussion

Imaging reveals a destructive mineralized lesion of the epiphysis and


metaphysis in a skeletally immature patient. Epiphyseal lesions include
chondroblastoma in skeletally immature patients and giant-cell tumors in
skeletally mature patients. The histology confirms the diagnosis with the
presence of multinucleated giant cells, “chicken- wire” calcifications, and
chondroid islands. Chondrosarcoma would look histologically more
aggressive than this and is unusual in children. Unicameral bone cysts do
not have mineralization and are typically confined to the metaphysis
radiologically. Eosinophilic granuloma can mimic many different lesions
radiographically and can occasionally be found in the epiphysis of
children, but the histology does not demonstrate classic Langerhans cells.
z

CLINICAL SITUATION FOR QUESTIONS 49 AND 50


Figure 49 is the radiograph of a 22-year-old woman with pain and snapping about the knee. Her
husband does not have this condition.
Question 49 of 100
What is the likelihood of this patient’s children having a similar condition?
1- No different than the likelihood among the general population
2- 25% only if her partner is similarly affected; if not, no different than the population average
3- 50%
4- 100%
Question 50 of 100
Germline alterations associated with this condition
1- affect proteins exostosin-1 (EXT1) and exostosin-2 (EXT2).
2- encode the alpha subunit of the stimulatory G-protein-coupled receptor, Gsa.
3- cause neoplastic cells to express vimentin, CD99, and FLI-1.
4- produce the WWTR1-CAMTA1 fusion.
z
discussion

The radiograph reveals multiple osteochondromas consistent with multiple hereditary


exostoses (also termed osteochondromatosis). This is inherited in an autosomal-dominant
fashion through the EXT1 and EXT2 genes. Thus, the likelihood of this patient’s children
being affected is 50% for each child. An autosomal dominant manner with germiline
alterations in EXT1 located at 8q24 and EXT2 located at 11p11-p12 occurs in
approximately 1 in 50,000 people.

Multiple hereditary exostoses is a disease in a spectrum of diseases caused by alterations


in either EXT1 or EXT2. The gene products of EXT1 and EXT2 genes are considered
essential for the function of fibroblast growth factor and Indian hedgehog signaling in the
normal growth plate and catalyze heparin sulphate polymerization. The G- protein-couple
receptor is involved in fibrous dysplasia, the expression of vimentin, and CD99. FLI-1 is
associated with Ewing sarcoma, and WWTR1-CAMTA1 fusion is associated with
epithelioid hemangioma, which is not related to multiple hereditary exostoses.
z

Figures 51a and 51b are the radiographs of an 83-year-old active, independent, and healthy woman who has
experienced 2 months of right lower thigh and knee pain. Her arthroplasty was previously well functioning, but
her pain has increased progressively for several weeks. While exiting a car she “bumped” her knee against the
door, felt a “crack,” and developed excruciating pain. She can no longer ambulate and was brought to the hospital.
Question 51 of 100
Based on imaging alone, what does this bone lesion most closely resemble?
1- Multiple myeloma
2- Implant-associated osteolysis
3- Metastatic renal cell carcinoma
4- Enchondroma
Question 52 of 100
Figures 52a through 52c show the biopsy of this lesion. Based on the clinical history, radiograph, and biopsy,
which diagnosis is most likely?
1- Enchondroma
2- High-grade osteosarcoma
3- Metastatic renal cell carcinoma
Question 53 of 100
z
Staging for patients with this diagnosis necessitates which study or studies?

1- CT scan of the chest and a bone scan

2- Skeletal survey
3- Bone marrow aspirate
4- Bone marrow biopsy

Question 54 of 100

Which local treatment option is most appropriate?

1- Radiation and wide surgical resection

2- Extended curettage with adjuvants


3- Wide surgical resection
4- Revision knee arthroplasty
z

discussion

This patient has a pathologic femur fracture. Her history of antecedent pain in the context of a previously well- functioning
implant suggests that a new process such as dedifferentiation of a long-standing lesion may have occurred. The initial
radiograph reveals a well-mineralized lesion within the intramedullary canal with punctate calcifications. This by itself suggests
an enchondroma or low-grade chondrosarcoma. It is important to note that enchondromas in the long bones rarely cause
pathologic fractures. This is not the case when they are present in the hands and feet, where enchondromas frequently have a
more aggressive radiologic appearance and pose higher risk. Radiographic findings concerning for malignant dedifferentiation
of an enchondroma include cortical thinning or breach, a soft-tissue mass, or periosteal elevation. The pathologic fracture
obscures the ability to identify these hallmarks of malignant degeneration.
Multiple myeloma, renal cell carcinoma, and osteolysis are all typically radiolucent. Implant-associated osteolysis
lesions are commonly multiple and periarticular. The histopathology reveals a cartilage tumor. The chondroid tissue appears to
be low grade. However, in an adjacent region there is a high-grade component seen both on the low-power view and the second
higher-power sample that does not resemble cartilage. This is highly suggestive of a dedifferentiated chondrosarcoma. This
component signifies that a portion of the original tumor (low-grade chondrosarcoma), which resembled the tissue of origin,
converted into a different cell lineage with more aggressive properties (nearly any type of high-grade sarcoma). Dedifferentiated
chondrosarcoma is a highly aggressive malignancy, with average 5-year survivals of less than 50%. As with most sarcomas, the
most likely site of metastatic dissemination of chondrosarcoma is pulmonary. Consequently, standard staging entails a high-
resolution CT scan of the lungs.
Serum protein electrophoresis and a skeletal survey are used for diagnosis of multiple myeloma. Positron emission
tomography/CT fusion scans are used at many centers in the staging of osteosarcoma, but their role in chondrosarcoma staging
is an emerging modality that is being investigated. Radiation is not typically used for local treatment of extremity
chondrosarcoma. Extended curettage is not appropriate for a high-grade lesion such as this. Revision arthroplasty alone does not
address the tumor. Wide resection is the local treatment of choice for high- grade chondrosarcoma; because of the fracture, this
patient may require an amputation to achieve wide margins.
z Question 55 of 100

Figures 55a through 55d are the MR images,


intraoperative photograph, and biopsy
specimen of a 33-year-old man with
progressive knee pain and recurrent
effusions. What is the most likely diagnosis?

1- Lipoma arborescens
2- Synovial osteochondromatosis

3- Pigmented villonodular synovitis (PVNS) 4-


Intra-articular hemangioma
z
discussion

MR imaging and gross and microscopic specimens are consistent


with PVNS. PVNS is a locally aggressive synovial tumor with 2 forms:
diffuse and nodular. Nodular PVNS occurs most commonly in the
hands, and diffuse is most common in the knee. Upon gross
examination, the diffuse form of PVNS is a tan mass of villi and folds
of synovium. Microscopically, PVNS is characterized by synovial cell
hyperplasia on the surface and below the synovium. Lipoma
arborescens would have imaging consistent with fat. Synovial
chondromatosis would show synovium laden with cartilage tissue,
and intra-articular hemangioma would show predominant vascular
channels.
z

Question 56 of 100

A disadvantage associated with presurgical (vs postsurgical) radiation therapy for


soft-tissue sarcoma is a

1- larger radiation field.


2- lack of formation of a “pseudocapsule” to facilitate resection. 3- higher local
recurrence rate.
4- higher wound complication rate.
z
discussion

Radiation therapy is used for soft-tissue sarcoma treatment to facilitate


resection of tumors that are close to adjacent structures and to
diminish risk for local recurrence. The disadvantages of radiation
therapy for soft-tissue sarcoma include patient inconvenience, risk for
secondary malignancy, and higher wound complication rates. The
advantages of presurgical (neoadjuvant) radiation therapy include a
smaller radiation field, formation of a “pseudocapsule” to facilitate a
planned close-margin resection, tumor shrinkage, and lower local
recurrence rates. The main disadvantage of presurgical radiation
therapy is the much higher risk for wound healing complications (as
demonstrated in numerous studies)
z

Question 57 of 100

Figures 57a and 57b are the close-up femur radiographs of a 73-year-old man with
nonsmall-cell carcinoma of the lung with visceral metastases. He has localized thigh pain
at rest and with any range of motion of the ipsilateral hip or knee. A bone scan shows
multiple areas of increased uptake. Other femur images in 2 views show no other lesions.
The medical oncologist predicts a survival of 3 months. What is the best next step?

1- Needle biopsy of the femur to confirm the diagnosis


2- Cephalomedullary femur nailing
3- Palliative radiation only
4- Curettage and cementation with plate fixation of the involved femur
z
discussion

A needle biopsy is not needed to confirm a diagnosis of metastatic disease when other sites of disease are
present. Cephalomedullary nailing offers a percutaneous option to prophylactically stabilize the impending
fracture and to assist in nursing and palliative care. Palliative radiation can be added, but risk for fracture
(even with complete bedrest) is nearly 100% according to Mirels’ criteria. An open procedure offers no
benefit to intramedullary nail placement. Recently, CT-based structural rigidity analysis has proven more
accurate than Mirel’s scoring for metastatic femoral lesion fracture prediction, but this analysis is unnecessary
when the lielihood of progression to fracture is so marked.
z

Question 58 of 100

Figures 58a through 58c are the radiograph, MR image, and sagittal CT scan of a 13-year-old boy with
a 5-month history of a left hip injury. He has no current pain, but substantial, progressive limitations in
hip flexion are present. What is the most likely diagnosis?

1- Extraskeletal osteosarcoma

2- Myositis ossificans
3- Chondrosarcoma
4- Pelvic osteosarcoma
z
discussion

This patient has a mineralized mass in the soft tissues in the rectus
femoris origin. The clinical history favors an avulsion injury with
subsequent myositis ossificans, and the imaging is consistent with
an ossified soft-tissue mass. Extraskeletal osteosarcoma is a rare
soft-tissue sarcoma and is generally seen in adults.
Chondrosarcoma is also generally seen in adults and would have a
more chondroid matrix as opposed to mature-looking bone on a CT
scan. Pelvic osteosarcoma could have this appearance on
radiograph, but the CT scan and MR image confirm this is a soft-
tissue process with mature mineralization.
z

CLINICAL SITUATION FOR QUESTIONS 59 THROUGH 61

Figures 59a through 59d are the CT scans and biopsy specimen of a 68-year-old man with progressive radicular symptoms
in his left lower extremity. Four years previously he underwent treatment of colorectal cancer with surgery, chemotherapy,
and radiotherapy.

Question 59 of 100

What is the diagnosis?

1- Radiation-associated insufficiency fracture 2- Postradiation osteosarcoma


3- Postradiation sclerosis
4- Metastatic colorectal cancer
Question 60 of 100
z
The best treatment is

1- chemotherapy and wide surgical resection.


2- bisphosphonate treatment with or without sacroplasty. 3- gabapentin or similar pharmacologic therapy.
4- radiation therapy.

Question 61 of 100

The prognosis for this condition is

1- influenced by the latent time to presentation.

2- generally favorable.

3- variable, with periodic flares and remissions.

4- relatively poor.
z
discussion

This patient has postradiation osteosarcoma of the sacrum after undergoing


treatment for pelvic cancer. Examinees should discern that this is a postradiation
sarcoma by the imaging findings of an osteoblastic tumor with extraosseous
extension arising in the expected radiation field for a patient who was treated with
radiation for colorectal cancer 5 years earlier. The histology demonstrates a high-
grade sarcoma.

Postradiation sarcomas are treated with chemotherapy and margin-negative surgical


resection, but they are associated with a relatively poor prognosis likely attributable to
the advanced age of most affected patients and the frequent axial location of these
tumors, which can delay diagnosis and make it difficult to obtain an appropriate
margin of resection. The imaging reveals a tumor, not an insufficiency fracture or
postradiation changes, and the histology shows a sarcoma, not colorectal cancer.
z

Question 62 of 100

Figures 62a and 62b are the radiographs of a 69-year-old woman who is seen for follow-up of a right arm
posterior compartment high-grade soft-tissue sarcoma. She previously underwent wide resection of the tumor
with preservation of the radial. She experienced an atraumatic fracture 2 years after completion of all treatment.
There is no evidence of recurrence on MR imaging. The patient desires treatment because of pain and instability
at the fracture site. What is the most reliable treatment option?

1- Injection of platelet-rich plasma and transcutaneous electrical stimulation


2- Debridement of the nonunion site, plate fixation of the fracture, and autologous bone grafting

3- Cemented long-stem proximal humerus replacement


4- Intramedullary humeral nail with bone morphogenetic protein-2 application
z
discussion

Cemented long-stem proximal humerus replacement is not dependent upon bone healing and will protect the remaining humeral diaphysis against subsequent fracture. Multiple
studies have demonstrated high failure rates

With attempted fixation of radiation-associated fractures. The radiated bone has low capacity for healing regardless of the fixation technique. Although bone graft, platelet-rich
plasma, growth factors, and electrical stimulation may have efficacy for other delayed unions and nonunions, their efficacy has not been reliably demonstrated for radiation-
associated fractures.
z

Question 63 of 100
Figures 63a through 63d are the radiograph, CT scan, MR image, and biopsy specimen of a 20-year-old rower who
has a several-month history of low-back pain. He has lost 10 pounds, but has no other constitutional symptoms. There is no bowel or
bladder incontinence, and he does not have neurologic symptoms. Which medication can be used to treat this condition?
1- Methotrexate
2- Nonsteroidal anti-inflammatory drugs (NSAIDs) 3- Denosumab
4- Adriamycin
z
discussion

Giant-cell tumor of bone (GCTB) is characterized by numerous multinucleated


osteoclast-type giant cells. Giant cells are known to express receptor activator of nuclear
factor ?B ligand (RANKL) and are responsible for the aggressive osteolytic nature of
tumors. Denosumab is a human monoclonal antibody that targets and binds with high
specificity to RANKL. Although generally benign, GCTB may be associated with multiple
local recurrences, multicentricity, pulmonary metastases, or lesions that cannot be
removed surgically without causing substantial morbidity. In a recent phase 2 study,
denosumab administered to patients with surgically salvageable and unsalvageable
GCTB was well tolerated and associated with inhibited disease progression (99%) and a
reduced requirement for surgery. Methotrexate and adriamycin are common
chemotherapeutic drugs used in the treatment of osteosarcoma of bone, but they have
shown no efficacy in the treatment of GCTB. NSAIDs are useful for treating pain
associated with osteoid osteomas, but they have no effect on GCTB.
z

Question 64 of 100

Which blastic metastases to bone are most common?

1- Breast and prostate carcinomas


2- Renal cell and thyroid carcinomas

3- Cervical and bladder carcinomas

4- Lung and liver carcinomas


z
discussion

Breast and prostate bone metastases are often blastic but may be
lucent or mixed. The other responses are predominantly lucent.
Algorithms to detect spinal metastases have been developed
because the spine and pelvis are the most common locations for
metastases
z

Question 65 of 100

Figures 65a and 65b are the axial and coronal MR images of a 72-year-old woman who
underwent a right knee arthroplasty 14 years ago. She has been referred by her primary
care provider for evaluation and management of a right medial knee mass. What is the
next best step?

1- Open surgical biopsy


2- Bone scan
3- Right knee radiographs

4- Staging CT scan of chest


z
discussion

The MR images reveal metal artifact from the knee arthroplasty and a
large mass medial to the joint line that has a relatively dark signal.
Because this patient had knee arthroplasty 14 years ago, a large
amount of polyethylene debris may have caused a pseudotumor
mass. A radiograph would reveal evidence of radiographic
wear/failure and is the best next step. If the patient needs revision
knee arthroplasty, a biopsy with frozen section could be performed at
the same time. Open biopsy might be appropriate, but further
information for pathologists regarding potential polyethylene wear
would be helpful. Embarking upon a staging workup at this point is
unnecessary and is not cost effective.
z

Question 66 of 100
Figures 66a and 66b are the clinical photographs of an 86-year-old man with a high-grade undifferentiated
pleomorphic sarcoma of the right thigh. He is pictured during preparations for wide surgical excision.
What is a favorable prognostic factor?
1- Superficial location with ulceration
2- Appendicular, lower extremity location
3- Histological evidence of extensive necrosis
4- Local inflammatory reaction to tumor
z
discussion

Improved survival is found in patients with soft-tissue sarcoma


involving the extremities (appendicular) rather than the torso (axial)
or head and neck. Although the superficial location above the
fascia is associated with improved survival, malignant ulceration or
fungation is an independent risk factor for poor outcome. Areas of
extensive necrosis are routinely found in high-grade
undifferentiated pleomorphic sarcomas and do not suggest a
favorable prognosis. Local inflammatory reactions to malignant
ulceration do not improve survival.
z

Question 67 of 100

Figures 67a and 67b are the fused positron emission tomography/CT scans of a
calf lesion at groin level in a 36- year-old woman with a soft-tissue mass. Her
biopsy specimen is shown in Figure 67c. Staging studies reveal no other lesions.
Using the American Joint Committee on Cancer (AJCC) staging system, the
stage is most likely

1- IA (G1 T2b N0 M0).

2- IIB (G1 T2b N0 M0).

3- III (G3 T2b N1 M0).

4- IV (G3 T2b N1 M1).


z
discussion

This patient has a high-grade subcutaneous angiosarcoma (G3


lesion) with subfascial extension as seen in Figure 67a and nodal
metastases as seen in Figure 67b (N1 [1 regional lymph node]).
Because there are no other lesions identified, there are no distant
metastases. Tumor size is difficult to determine without a full
imaging study, but all responses are T2b (tumors >5 cm with deep
extension). This is a stage III tumor according to the AJCC system.
z
z

Question 68 of 100

Figure 68a is the clinical photograph of a 59-year-old woman who has had a long-standing
fungating ulcer on her left lower leg. She states that the ulcer began as a small reddened
area and gradually enlarged during the last 4 years. Anteroposterior (AP) and lateral
radiographs of her left leg are shown in Figures 68b and 68c. A whole-body bone scan is
shown in Figure 68d. An axial T1-weighted MR image is shown in Figure 68e. A CT scan of
the pelvis at the level of the groin is shown in Figure 68f. A histologic specimen is shown in
Figure 68g. Based on the clinical, radiographic, and histologic information, the diagnosis is

1- poorly differentiated squamous carcinoma.

2- dermatofibrosarcoma protuberans.
3- melanoma.
4- locally advanced osteomyelitis.
z
discussion

The clinical photograph reveals a large ulcerated lesion of the anterior leg. The AP and lateral
radiographs reveal the soft-tissue abnormality, and the bone scan reveals increased metabolic activity
of the underlying tibia. T1- weighted MR imaging reveals the lesion wrapping around the anterior tibia
with loss of subcutaneous tissue. A CT scan of the pelvis reveals an inguinal lymph node. The
histology reveals nests of polyhedral cells surrounded by reactive fibrosis. The diagnosis is poorly
differentiated squamous cell carcinoma, and the patient has the characteristic findings of a long-
standing squamous cell carcinoma with poor differentiation and likely regional lymph node metastasis.
The history of a small reddened area does not suggest any of the other diagnoses. Although keratin
pearls are not shown in this histologic field, nests of polyhedral cells indicate squamous cell
carcinoma. A small subset of patients with squamous cell carcinoma will have advanced disease. Size
and differentiation, as well as type of surgical procedure and margins of resection, are of prognostic
significance. Sentinel node evaluation for patients at high risk has been suggested, including
evaluations for lesions that are larger in size and with poor differentiation, perineural invasion, or
compromised immunologic states. The nonpreferred responses are not associated with the clinical
presentation of squamous cell carcinoma. Squamous cell carcinomas are keratin positive on
immunohistochemistry.
z

CLINICAL SITUATION FOR QUESTIONS 69 AND 70

Figure 69a is the radiograph of a 39-year-old woman with metastatic lung cancer who underwent a prophylactic right intramedullary nail procedure.
There were no intraoperative or immediate postsurgical problems. Ten hours after surgery, the patient became increasingly confused and agitated. An
examination was notable for altered mentation, tachycardia, and new-onset hypoxemia. Chest radiographs were obtained before surgery (Figure 69b)
and immediately after symptom onset (Figure 69c).

Question 69 of 100

Which treatment will most likely improve this patient’s condition?

1- Broad-spectrum intravenous antibiotics

2- High-dose corticosteroids
3- Resuscitation and oxygen
4- Anticoagulation with heparin
z

Question 70 of 100

Which prophylactic measures may decrease risk for this


postsurgical complication?

1- Distal femoral venting, perioperative hydration, and oxygenation


2- Presurgical antibiotics and antibiotic redosing
3- Perioperative normothermia with a forced-air warming device
4- Neuraxial or regional anesthesia
z
discussion

This patient’s history, examination, and imaging findings are most consistent with fat embolism syndrome
(FES). Classic findings of FES are mental status changes, hypoxemia, and petechial rash developing
after intramedullary instrumentation procedures. FES is more common after closed femoral nailing for
cancer treatment than after traumatic fractures. Patients with preexisting cardiopulmonary dysfunction
are at increased risk. FES remains a clinical diagnosis, although a high-resolution CT scan may
distinguish fat emboli from thrombi, and diffusion- weighted brain MRI may demonstrate fat emboli.
Aggressive supportive care including as-needed mechanical ventilation and fluid resuscitation remains
the primary treatment. A 2009 meta-analysis suggested that prophylactic steroids reduce the incidence
and severity of FES, but evidence supporting the use of corticosteroids for established FES is anecdotal.

FES prophylaxis generally consists of intraoperative hydration and oxygenation. Some authors suggest
that distal femoral venting is beneficial to decrease intramedullary pressure and subsequent fat
embolism. Broad-spectrum antibiotics would be an appropriate treatment for aspiration pneumonia. This
patient has diffuse bilateral infiltrates as opposed to a focal infiltrate. Furthermore, the mental status
changes are more consistent with FES than pneumonia. Anticoagulation would be the treatment of
choice for pulmonary embolism. The timing after surgery is more consistent with FES, as are the
changes in mental status.
z

Question 71 of 100
Figures 71a through 71d are the radiographs, MR images, and biopsy specimen of a 15-year-old boy with a several-
month history of right hip pain with no history of injury. This condition is associated with increased activity of which gene
product?
1- FGFR3
2- Ga stimulatory protein 3- COMP
4- EXT-1
z
discussion

Fibrous dysplasia is a common benign skeletal lesion that may involve 1 bone
(monostotic) or multiple bones (polyostotic) and occurs throughout the skeleton with a
predilection for the long bones. The radiographic features of fibrous dysplasia
typically illustrate a grayish “ground-glass” pattern that is similar to the density of
cancellous bone. The key histologic features of fibrous dysplasia are trabeculae of
immature bone, with no osteoblastic rimming, contained within a bland fibrous stroma
of dysplastic spindle-shaped cells without any cellular features of malignancy. The
etiology of fibrous dysplasia has been linked to an activating mutation in the gene that
encodes the a subunit of stimulatory G protein located at 20q13.2-13.3. This leads to
a constitutive activation of adenylate cyclase and increased cyclic adenosine
monophosphate formation. FGFR3 mutations are associated with achondroplasia.
COMP mutations are associated with pseudoachondroplasia and multiple-epiphyseal
dysplasia. EXT-1 mutations are associated with multiple hereditary exostosis
z
Question 72 of 100

Figures 72a through 72d are the radiograph, MR images, and


biopsy specimen of a 42-year-old man with an insidious onset of
left hip pain. Further imaging reveals no other lesions. What is the
most appropriate initial treatment?

1- Bisphosphonate or denosumab and observation with


cementoplasty for refractory pain

2- Chemotherapy and wide surgical resection


3- Wide surgical resection alone
4- Extended intralesional curettage and grafting
z
discussion

This patient has a localized pelvic chondrosarcoma. Treatment is


wide surgical resection. There is no defined role for chemotherapy
or radiotherapy in the setting of conventional chondrosarcoma.
Additionally, while intralesional treatment may be used for select
low-grade extremity chondrosarcomas, it is not indicated for axial
lesions. Treatment involving less than a wide surgical margin
correlates with local recurrence.
z

Question 73 of 100

Figures 73a through 73c are the radiographs of a 68-year-old woman with a pathologic
left femur fracture. A clinical examination demonstrates a large soft-tissue mass at the
fracture site. CT scans of the chest, abdomen, and pelvis reveal numerous enlarged
lymph nodes. Frozen section analysis at open biopsy reveals relapsed lymphoma. What
is the most appropriate treatment?

1- Antegrade reconstruction nail


2- Antegrade reconstruction nail and distal femur plate fixation

3- Retrograde femoral nail


4- Retrograde femoral nail and proximal femur plate fixation
z

discussion
Retrograde femoral nail fixation is mechanically advantageous for this
fracture because it permits fixation distal to that which can be achieved with
an antegrade nail. Most modern retrograde nail systems permit placement of
multiple distal interlock screws that can be locked in a fixed-angle construct.
Lytic bone destruction extends to the
femoral condyles, making antegrade femoral nail fixation comparatively
unsuitable. Classic orthopaedic oncology principles advise fixation of the
entire bone whenever feasible. Evidence indicates that new distant
metastasis within a fixated femur is a rare event, and efforts to protect the
“whole bone” may not be warranted, especially if they entail additional risk.
This argument is stronger regarding lymphoma-associated fractures (vs the
more common carcinoma-associated fractures) because chemotherapy and
radiation may be used to treat lymphoma with curative rather than palliative
intent. Combined plate and nail fixation is not preferred in this scenario.
Plate fixation alone or augmented with cement is a viable option for this this
fracture; however, the presence of a large soft tissue would likely necessitate
extensive tumor debulking to appropriately place the plate.
z

Question 74 of 100

Survival estimation in guiding surgical decision-making in


metastatic spine disease includes scores that include

1- visceral metastases, multiple bone metastases, and tumor types


with poorer prognosis.

2- body mass index, lower hemoglobin levels, older age, and


visceral metastases.
3- peripheral vascular disease, diabetes, and visceral metastases.
4- disease confined to only adjacent vertebrae.
z
discussion

The modified Bauer and 6 other scoring systems have been used as accurate
measures with which to establish the prognosis of metastatic spine disease.
The modified Bauer scoring system assigns 1 point for no visceral
metastases; 1 point if the diagnosis is not lung cancer; 1 point if the primary
tumor is breast, kidney, lymphoma, or multiple myeloma; and 1 point if the
tumor is a solitary skeletal metastasis. If the score totals 0 to 1, the
recommendation is supportive care and no surgery. If the prognostic score is
2, short-term palliative surgery with a dorsal approach is performed. If the
score is 3 to 4, middle-term local control with both ventral and dorsal
approaches is suggested. The decision for or against surgery should not be
based alone on any prognostic score but should take symptoms and
neurological compromise into account.
z

Question 75 of 100

A 52-year-old woman with a medical history that includes type 1 diabetes mellitus and rheumatoid arthritis
has a painless right thigh mass that increased in size during the preceding year. Ultrasound was “consistent
with lipoma,” and the patient underwent uneventful resection. Final pathology revealed high-grade
undifferentiated sarcoma. Figures 75a and 75b are the clinical photograph and postresection MR image. The
treatment rendered prior to referral to a sarcoma center most likely will result in increased

1- likelihood for amputation.

2- likelihood for flap coverage.

3- overall mortality.
4- need for radiation therapy.
z

discussion

This patient had an unplanned resection of a high-grade soft-tissue


sarcoma. The MR image shows that the unplanned resection extended
deep to the fascia. Errors in this case include failure to obtain cross-
sectional imaging of a tumor deep to the fascia prior to resection and use
of a transverse incision. Flap coverage for unplanned soft- tissue sarcoma
resection can increase the complexity of soft-tissue reconstruction.
Radiation therapy would have
been indicated for a high-grade soft-tissue sarcoma deep to the fascia
regardless of the biopsy technique. Overall, mortality does not correlate
with errors in biopsy technique. Although many studies demonstrate
increased local recurrence risk is associated with unplanned resection,
amputation is not indicated in most cases. Radiation therapy and wide re-
resection with salvage of the involved limb is the treatment of choice.
z

Question 76 of 100

A 16-year-old boy with a high-grade conventional osteosarcoma of the right proximal tibia has completed neoadjuvant
chemotherapy. A restaging radiograph and MR image are shown in Figures 76a and 76b. Wide resection with limb salvage is
planned. Which muscle will provide the primary protective margin for the tibial nerve and popliteal vessels?

1- Soleus

2- Popliteus

3- Medial gastrocnemius

4- Lateral gastrocnemius
z
discussion

The popliteus muscle originates from the posterior aspect of the


proximal tibia and serves as the primary barrier to tumor
encroachment upon the tibial nerve and popliteal vessels. The
medial and lateral heads of the gastrocnemius lie superficial to the
neurovascular bundle. A portion of the soleus originates from the
lateral proximal tibia, forming part of the soleal arch through which
the neurovascular bundle passes; however, the majority of the
soleus is superficial to the neurovascular structures
z

Question 77 of 100

Figures 77a and 77b are the recent knee radiographs of a 53-year-old man whose history includes tobacco use and
secondary polycythemia. He is now experiencing bilateral knee pain, knee swelling, and increasing discomfort with
ambulation. All efforts at nonsurgical treatment have failed. What is the most reasonable next treatment option?

1- Core biopsy
2- Knee arthroplasty with postsurgical radiation therapy

3- Knee arthroplasty
4- Debridement with intravenous antibiotics
z
discussion

The radiographs reveal bilateral bone infarcts with subchondral


collapse. The images are diagnostic for bone infarct. Other
hematological conditions are associated with multiple bone infarcts,
including sickle-cell disease, hemophilia, aplastic anemia,
thalassemia, and acute lymphoblastic leukemia. Postsurgical
radiation therapy is considered for Paget disease, but the
radiographic appearance is not consistent with that diagnosis.
z

CLINICAL SITUATION FOR QUESTIONS 78 AND 79


Figures 78a through 78d are the radiograph, CT scans, and biopsy specimen of a 45-year-old man with a history of treatment for
localized low-grade pelvic chondrosarcoma with limb salvage. He now has recurrent pain about his
hip.
Question 78 of 100
This patient should be told that
1- he has developed a prosthetic joint infection with purulence on biopsy.
2- he has developed a metal-on-metal pseudotumor reaction.
3- the tumor has recurred with transformation to a higher grade.
4- the tumor has recurred in a similar manner.

Question 79 of 100
Definitive treatment for this tumor likely will necessitate
1- irrigation and debridement with single vs 2-stage prosthetic exchange and intravenous antibiotics.
2- chemotherapy and radiotherapy.
3- hemipelvectomy.
4- revision to a metal-on-highly crosslinked polyethylene construct.
z
discussion

This patient has a recurrent tumor after limb salvage resection for
low-grade chondrosarcoma. In addition to recurrence, the biopsy
specimen shows transformation to a higher-grade chondrosarcoma
(but not a dedifferentiated chondrosarcoma). There is no evidence
of infection or a metal-on-metal–related pseudotumor.

Treatment that results in the highest likelihood of cure involves


wide/radical resection of the area and frequent conversion to
external hemipelvectomy. There is no defined role for
chemotherapy or radiotherapy in the treatment of conventional
chondrosarcoma, even when it is high grade.
z

Question 80 of 100. Figure 80a is the radiograph of a 5-year-old girl who has experienced 8 weeks of
shoulder pain that is mostly relieved with nonsteroidal anti-inflammatory drugs. Her MR images are shown
in Figures 80b through 80f. Histology is shown in Figures 80g through 80i. The most likely diagnosis is
1 aneurysmal bone cyst
2 ewing sarcoma
3- giant-cell tumor.
z
discussion

Osteoblastoma most often is diagnosed in adolescents and young adults, with 75% of patients
younger than 25 years of age at diagnosis. Within tubular bones, osteoblastoma most often arises
in the metadiaphyseal region, but it is more commonly seen in the spine within the posterior
elements and flat bones. The lesion is well defined on radiographs, with lucent to mixed-lucent
and blastic areas. MR imaging findings reveal low to intermediate–signal intensity on T1-weighted
images and intermediate to high–signal intensity on T2- weighted images. Neoplastic woven bone
can be seen in Figure 80g, and osteoclasts are shown in Figures 80h and 80i. Although the bone
is expansile in appearance on radiologic studies, there are no large vascular lakes on histology to
support the diagnosis of an aneurysmal bone cyst. Although there are areas of woven bone as in
fibrous dysplasia, the appearance of osteoblastic rimming rules against this diagnosis. The
location and histology of the tumor are inconsistent with giant-cell tumor of bone, the latter being a
metaepiphyseal tumor histologically comprising giant cells within a background of stromal cells
having nuclei identical to those of the giant cells. Ewing sarcoma would be a diagnostic
consideration for a destructive scapular lesion in a 5-year-old, but the histology does not
demonstrate the small, round blue-cell morphology of an Ewing sarcoma.
z

Question 81 of 100
A 32-year-old man underwent distal femur resection and endoprosthetic replacement at age 15 for high-grade conventional
osteosarcoma. He was treated with neoadjuvant and adjuvant cisplatin, doxorubicin, and methotrexate. There has been no evidence of
recurrent osteosarcoma, and he has been otherwise active and well. He is scheduled to undergo exchange of the polyethylene liner and
bushings in his prosthesis because of wear that has caused recurrent effusions and a sensation of instability. Which study is most
important to assess his perioperative medical risk?
1- Pulmonary function tests
2- 24-hour creatinine clearance
3- Hepatic transaminases
4- Echocardiogram
z

DISCUSSION

This patient was treated with doxorubicin, an anthracycline chemotherapy agent that
causes cardiomyopathy in a dose-dependent fashion. Patients with osteosarcoma
frequently undergo noninvasive cardiac testing with an echocardiogram or radionuclide
angiography prior to anthracycline treatment. Cardiac toxicity is the typical cumulative
dose-limiting effect of these agents. Posttreatment cardiac surveillance testing with an
echocardiogram or radionuclide angiography is recommended for childhood cancer
survivors treated with anthracyclines. Renal toxicity is a common adverse effect of
cisplatin but usually manifests acutely. Similarly, methotrexate may cause acute
hepatic toxicity that manifests acutely. In contrast, congestive heart failure attributable
to doxorubicin cardiotoxicity frequently manifests late, with patients typically
experiencing no or minimal symptoms until substantial cardiac dysfunction has
occurred. Areflexia and peripheral neuropathy are also common side effects
associated with these chemotherapeutic agents
z

Question 82 of 100

Figures 82a and 82b are the histopathology of an


otherwise healthy 50-year-old man who had a growing
mass marginally excised from his shoulder with local
anesthetic and no presurgical imaging. The mass
includes the deep fascia and muscle and is larger than
5 cm. Postsurgical MR imaging was obtained, and T2
and T1+C images are shown in Figures 82c and 82d.
What is the best local treatment option?

1- Observation
2- Radiation only
3- Chemotherapy only
4- Reexcision and radiation
z
discussion

The misdiagnosis of a soft-tissue mass is fairly common. However, in this scenario, the first
indication that the presumed diagnosis was incorrect is supported by rapid growth over a short time
period. Generally, lipomas and many other benign soft-tissue masses grow slowly over years rather
than weeks or months. On MRI, a lipoma classically demonstrates homogenous high-signal intensity
on a T1 sequence and fairly low-signal intensity on a T2 sequence. No imaging was done before
surgery, but a high-grade sarcoma generally shows a heterogenous signal on T1 and T2 sequences
that is distinctly different than the pattern of a lipoma. The histopathology is consistent with a high-
grade sarcoma; a spindle-cell tumor with hypercellularity, anaplasia, and pleomorphism. Mitotic
figures are also noted, suggesting rapid turnover and growth. This is a high-grade sarcoma that is
treated with wide surgical excision, and observation is not recommended. External beam radiation is
often used as an adjuvant either before or after surgery to help decrease the local recurrence rate.
Radiation alone would not be sufficient treatment. The use of chemotherapy is controversial. In the
setting of a “whoops” or unintended resection of a high-grade soft- tissue sarcoma, reexcision of the
tumor bed with radiation is generally indicated. The radiation is used to eliminate microscopic
contamination. However, evidence supports surgery alone for superficial (subcutaneous) sarcomas.
z

CLINICAL SITUATION FOR QUESTIONS 83 THROUGH 86

Figures 83a through 83d are the prefracture and postfracture radiographs of a previously healthy 19-year-old man who learned he had a high-
grade conventional osteosarcoma of his left distal femur after undergoing open biopsy of a large left thigh mass. Staging study findings were
negative for evidence of metastases, and he began neoadjuvant chemotherapy. Despite use of a hinged knee brace and weight bearing
limitations with crutches, he experienced a sudden increase in pain while rising from bed between his first and second cycles of chemotherapy.

Question 83 of 100

The optimal initial treatment of this pathologic fracture is

1- spanning external fixation with continuation of neoadjuvant chemotherapy.


2- above-knee amputation with postsurgical adjuvant chemotherapy determined by the histologic findings from the specimen.
3- cast immobilization of the fracture with continuation of neoadjuvant chemotherapy.
4- immediate limb-sparing resection and endoprosthetic reconstruction with adjuvant chemotherapy determined by histologic findings from the
specimen.
Question 84 of 100 z

A series of axial T1-weighted MR images is shown in Figure 84. The postfracture MRI reveals tissue contamination by fracture hematoma.
Based upon the MRI findings, which aspect of limb-sparing resection and reconstruction will be most challenging?

1- Preserving sciatic, tibial, and common peroneal nerve function

2- Obtaining adequate fixation of the distal femur component


3- Soft-tissue coverage of the endoprosthesis
4- En bloc extra-articular resection of the knee joint

Question 85 of 100

The patient ultimately undergoes rotationplasty. What is the most likely etiology of early limb loss after rotationplasty for sarcoma?

1- Vascular compromise

2- Wound infection
3- Local recurrence
4- Fixation failure

Question 86 of 100

When compared to patients who undergo endoprosthetic reconstruction, patients who undergo rotationplasty are more likely to

1- have difficulty obtaining gainful employment.


2- participate in high-impact exercise and sports.
3- require revision surgery at some point in their lifetime.

4- experience problems with psychosocial adjustment.


z
discussion
Debate exists regarding the prognostic significance of pathologic fractures with respect to both overall survival and local recurrence,
with most older studies indicating that fracture increases these risks and more recent research suggesting that risks are not increased as
much as previously believed. This patient’s fracture is minimally displaced and amenable to cast immobilization. Healing of
osteosarcoma pathologic femur fractures treated with casting and neoadjuvant chemotherapy has been well documented. Acute
amputation was historically the standard treatment and remains an option for markedly displaced fractures with severe contamination.
However, this scenario remains amenable to limb salvage. Spanning external fixation is suboptimal because the femoral canal will be
breached proximally, creating a potential site of infection or a fracture stress riser if a stemmed endoprosthesis is to be used. Immediate
limb-sparing resection is not advised because there will be higher risk for contamination
from the acute hematoma, which will likely resolve with casting and chemotherapy.
The MRI series reveals extensive contamination of the knee joint by fracture hematoma. Such contamination precludes the more
commonly performed intra-articular resection. Extra-articular resection, although technically feasible, is more difficult and frequently
less functional than intra-articular resection. The MRI reveals that the sciatic nerve and its branches are distant from the fracture
hematoma and tumor. Femoral fixation is not challenging in this patient because the tumor is quite distal. Soft-tissue coverage should be
adequate because no subcutaneous contamination is shown on MRI.
Rotationplasty is a reasonable alternative to extra-articular resection and amputation among adults. Only the major nerves need to be
free of tumor because even contaminated vessels can be resected with subsequent anastomosis after rotation and shortening. Vascular
compromise is the most common cause of early limb loss after rotationplasty because both anastomosed and rotated vessels are at risk
for thrombosis. Local recurrence is typically a later complication. Wound infections and fixation failures may occur but rarely result in
acute limb loss.
Patients who undergo rotationplasty function at a level comparable to those who undergo below-knee amputation. High-impact
activities are not discouraged for patients who undergo rotationplasty because there are no concerns about prosthetic wear or loosening.
Psychosocial and occupational impairment are not worse among patients who undergo rotationplasty vs other forms of reconstruction.
Revision surgery is less common with rotationplasty because there are fewer late infections, and concerns about prosthetic wear and
loosening are eliminated.
z

Question 87 of 100

Figures 87a through 87d are radiographs of both knees of a 75-year-old man who is experiencing bilateral
knee pain and stiffness. The gross appearance of the lesions in the suprapatellar pouch is seen in Figure 87e.
The histologic finding that would best determine this process is osteoarthritic in nature and not neoplastic is
the absence of

1- chondrocyte cloning.

2- synovial metaplasia.

3- cords of eosinophilic cells in myxoid stroma.


z
discussion

The radiographic appearance is typical for loose bodies caused by


osteoarthritis. The process is bilateral, which is not characteristic of
synovial chondromatosis. The characteristic histologic finding in synovial
chondromatosis is synovial metaplasia, the cartilaginous change of the
synovial membrane. Chondrocyte cloning is a finding in chondrosarcoma.
Cords of eosinophilic cell in myxoid stroma are characteristic for
extraskeletal chondrosarcoma. Histologic evaluation is not necessary to
rule out synovial chondromatosis because of the characteristic radiographic
findings, although it is not unreasonable to document the specimens
removed with a pathology consultation. In synovial chondromatosis, bone
morphogenetic proteins are thought to be involved in the pathogenesis of
the disease.
z

Question 88 of 100

Figures 88a through 88d are the radiographs and biopsy specimens of a 65-year-old
woman with a history of breast cancer who has been experiencing 6 weeks of increasing
left hip pain. She denies any injury. What is the most likely diagnosis?

1- Metastatic breast cancer

2- Myeloma
3- Fibrous dysplasia
z
discussion

Radiographs reveal a destructive lesion. A differential diagnosis would include metastatic


disease, myeloma, lymphoma, or primary sarcoma of bone. Histology shows sheets of plasma
cells with eccentric nuclei, coarsely clumped nuclear chromatin, and a perinuclear halo. This
pathology is consistent with myeloma. Metastatic disease would reveal nests of epithelia cells in
a fibrous background. Fibrous dysplasia would reveal irregular bony trabecular without rimming
osteoblasts. Osteosarcoma would reveal malignant cells making osteoid.
z
z

Question 89 of 100

Figures 89a through 89g are the radiographs, MR images, and biopsy
specimen of a 32-year-old man who has chronic left thigh and knee pain
and recent knee swelling with no specific trauma or injury. The pain is
exacerbated with activity but is also present at rest and at night. What is
the most appropriate treatment?

1- Surgical wide resection and endoprosthetic reconstruction


2- Chemotherapy followed by wide surgical resection and reconstruction
3- Aggressive curettage with allograft placement
4- Aggressive debridement, antibiotic bead placement, and intravenous
(IV) antibiotics
z
discussion

The radiographs show a lytic, destructive, poorly marginated femur lesion.


MR imaging confirms the infiltrative nature of the lesion, a communication
with the joint, and a substantial nonenhancing joint effusion. Biopsy confirms
numerous polymorphonuclear neutrophils consistent with osteomyelitis with
septic arthritis. The treatment of femur osteomyelitis with a septic knee
includes aggressive debridement of all nonviable tissue, placement of
antibiotic beads, and IV antibiotics. Surgical wide resection and
endoprosthetic reconstruction would be considered only if the lesion were a
low-grade malignancy. Aggressive curettage with allograft placement would
be considered for benign tumors. Allograft placement is not appropriate in the
setting of an infection. Chemotherapy followed by wide surgical resection and
reconstruction is only appropriate for a high-grade malignancy.
z

CLINICAL SITUATION FOR QUESTIONS 90 AND 91

Figures 90a and 90b are the axial MR image and histology of a 27-year-old woman who
has a firm lesion in the popliteal fossa. Figure 90c is the gross specimen after resection.

Question 90 of 100

What is the most likely diagnosis?

1- Desmoid tumor
2- High-grade pleomorphic sarcoma

3- Synovial sarcoma
4- Extraskeletal Ewing sarcoma
z

Question 91 of 100

Immunohistochemical analysis of this lesion will be characterized


by expression of

1- CD99.
2- epithelial membrane antigen.

3- estrogen receptor.
4- S100.
z
discussion

The imaging shows an infiltrative soft-tissue mass. Histology


demonstrates a bland spindle-cell lesion without nuclear atypia
consistent with desmoid tumor. Extraskeletal Ewing sarcoma would
be expected to reveal more surrounding edema and small, round
blue cells on histology. Synovial sarcoma would have a biphasic
appearance consisting of spindle cells and plump epithelial cells
arranged in glands or cords. Epithelial membrane antigen is often
found in synovial sarcoma. Studies have demonstrated that
desmoid tumors express estrogen receptors. As a result, tamoxifen
has been 1 of the modalities used to treat this lesion.
z

Question 92 of 100

Figures 92a and 92b are the reconstruction radiographs of a 16-year-


old boy with a high-grade conventional osteosarcoma of his right
proximal tibia. He has completed neoadjuvant chemotherapy. Wide
resection, endoprosthetic proximal tibia replacement, a medial
gastrocnemius flap, and a split-thickness skin graft were performed.
The preferred postsurgical knee rehabilitation regimen is

1- therapist-directed passive and active assisted range of motion


exercises from 0 to 90 degrees.
2- immobilization in full extension for 6 weeks with therapist-directed
gait training and isometric exercises.
3- use of an immediate continuous passive motion machine set at 0
to 90 degrees followed by outpatient physical therapy.
4- use of a knee immobilizer for 2 weeks followed by home use of a
continuous passive motion machine (set at 0 to 90 degrees).
z
discussion

This patient underwent resection of the patellar tendon insertion onto the
proximal tibia. The residual tendon is sutured to the prosthesis with
augmentation of the repair by the medial gastrocnemius flap. Immobilization for
6 weeks in full extension is preferred to permit healing of the tendon repair and
prevent patella alta with associated extensor lag. Cast, splint, or brace are all
acceptable options for the initial immobilization. Risk for knee stiffness with such
immobilization is low because all of the distal knee ligamentous and capsular
attachments are transected with resection. Two weeks of immobilization may be
adequate for flap and skin graft healing, but this amount of time is inadequate
for patellar tendon healing. Continuous passive motion vs therapist-directed
range of motion has not been rigorously compared for proximal tibia
replacement. Responses 1 and 2 include early motion.
z

Question 93 of 100
Figures 93a through 93f are radiographs, selected MR imaging sequences, and biopsy specimens of the left humerus of a 76-year-old
woman who has experienced long-term left arm pain. She has received previous treatment for osteoarthritis of her left shoulder with
nonsteroidal anti-inflammatory drugs and an intra-articular corticosteroid injection for her rotator cuff arthropathy. Recent staging
studies show no evidence of metastatic disease. What is the most appropriate next treatment?
1- Neoadjuvant chemotherapy followed by wide resection 2- Presurgical radiation therapy followed by wide resection 3- Wide
resection and endoprosthetic reconstruction
4- Systemic chemotherapy and local radiation
z
discussion

This patient’s radiographs, MR imaging, and histologic examination are


diagnostic for low-grade chondrosarcoma, which is treated with surgery
alone. In this patient, the tumor is entirely intramedullary. Both wide
resection and extended intralesional curettage with intraoperative
surgical adjuvant treatment are treatment options. Chemotherapy and
radiation are not beneficial in the treatment of low-grade
chondrosarcoma.

Endoprosthetic reconstruction using a reverse shoulder arthroplasty may


be employed when the rotator cuff is deficient for older patients with
neoplasms of the proximal humerus. Allograft-prosthetic composite
reverse shoulder arthroplasty also has been used with early success.
z

Question 94 of 100
Figures 94a through 94f are the radiographs and MR images of a 16-year-old boy who
experiences left ankle pain with activity. What is the most likely diagnosis?
1- Aneurysmal bone cyst (ABC)
2- Fibrous dysplasia
3- Simple bone cyst
4- Ewing sarcoma
z
discussion

Radiographs reveal a purely lytic, mildly expansile lesion of the distal fibula
metaphysis in a skeletally immature patient. All of the responses can be seen in
the fibula of skeletally immature patients and can show lytic areas. The MR
images show multiple fluid-fluid levels in the lesion consistent with an ABC lesion.
Simple bone scans can demonstrate a single fluid-fluid level particularly if they
have fractured, but are generally less expansile and do not have the multiple fluid-
fluid levels seen with ABCs. Ewing sarcoma and fibrous dysplasia tend to be more
diaphyseal in location and do not involve the fluid-fluid levels seen in this patient.
An aneurysmal bone cyst can occur because a secondary lesion is almost any
bone tumor. Telangiectatic osteogenic sarcoma may mimic ABC and should be
considered. Surgeons should perform biopsies with the highest-grade lesion
considered in the differential and confirm pathology prior to proceeding with
definitive treatment.
z

Question 95 of 100 A patient undergoes excision of a presumed


lipoma of the superficial thigh. Final pathology reveals synovial
sarcoma without reference to the margins. What is the
recommendation for definitive treatment?

1- Observation
2- External beam radiation therapy alone
3- Wide resection of the tumor bed and radiation therapy

4- Radiation therapy and chemotherapy


z
discussion

An unplanned excision of a soft-tissue sarcoma occurs when a surgeon removes a soft-tissue mass while believing it is benign, and,
upon pathologic examination, learns that it is malignant. These procedures often are performed without presurgical staging studies or
appropriate oncologic surgical technique. Following an unplanned excision,
patients usually are referred to a sarcoma center for definitive care. Definitive treatment is hindered by the lack of presurgical imaging,
sophisticated pathologic examination findings, and a contaminated tumor bed larger than the original tumor. Further, as many as 50%
of cases are associated with residual disease within the tumor bed, even when no tumor can be identified with imaging or an
examination and the initial surgical report indicates all gross disease has been removed. Most patients for whom an inadequate
excision of an unsuspected sarcoma is performed are treated with reexcision of the tumor bed, and, very often, adjuvant radiotherapy.
z

Question 96 of 100
Figures 96a and 96b are the MRI sections of the symptomatic left knee of a 28-year-old man with left anterior knee
pain 18 months after undergoing an allogenic bone marrow transplant for acute myelogenous leukemia. His
intraoperative fluoroscopy images are shown in Figures 96c and 96d. What is most critical when obtaining a diagnosis
for this patient?
1- Sampling from the periphery of the lesion
2- Immediate formalin fixation
3- Sending unfixed fresh tissue
4- Fixation with ribonucleic acid (RNA) stabilization solution
z
discussion

Flow cytometry and cytogenetic analysis are frequently crucial to make an


accurate diagnosis of leukemia and lymphoma. These techniques are best
performed on fresh, unfixed tissue. Optimal technique includes moistening
the tissue to prevent desiccation and immediately sending the specimen for
pathologic analysis. Communication with the pathologist regarding the
possibility of a leukemia or lymphoma is prudent to facilitate prompt
processing. It is advisable to have a hematology technician in the operating
room to handle tissue. Sampling the periphery of tumors with cystic
components is often necessary to obtain cellular tissue, but this necessity is
not specific to leukemia and lymphoma cases. Formalin is the standard tissue
preservation solution for most biopsies. RNA stabilization solution is usually
used for research rather than clinical purposes.
z

Question 97 of 100

Figures 97a through 97d are the radiographs and MR images of a 21-year-old man with symptoms of a
left medial thigh mass. Upon examination, you palpate a firm, fixed, deep, nontender mass of the medial
proximal left thigh. No other masses are found during the examination. The patient fears metastatic
disease. What is the risk for malignant transformation throughout this patient’s lifetime?

1- 0%
2- Less than 1%

3- Less than 15%

4- More than 15%


z
discussion

The images reveal a solitary pedunculated osteochondroma.


Malignant degeneration of solitary osteochondromas occurs in
fewer than 1% of patients. A reasonable approach would be to
inform the patient of the rare (less than 1%) incidence of malignant
degeneration and to return for evaluation if symptoms develop or if
the patient notices growth of the lesion.
z

Question 98 of 100

Figures 98a and 98b are the radiograph and biopsy specimen of a 20 year old man who is being evaluate for the
first time for foot pain. Treatment shoud include

1- first-ray amputation

2- extended curettage and grafting or cementation

3- extended curettage, radiation, and chemotherapy

4- wide resection and chemotherapy.


discussion
z
This pathology is most consistent with
giant-cell tumor. Note the presence of
multinucleated cells and stroma of
spindlelike cells with pale staining
cytoplasm and nuclei. Giant-cell tumors
typically occur in patients ages 20 to
40. Common sites include the epiphysis
of the distal femur or proximal tibia
(50% of the time). Although it is a
benign lesion, giant-cell tumors have a
tendency for bone destruction,
recurrence, and, rarely, metastasis. The
initial
treatment of choice is curettage with
grafting or cementation. For recurrent
or stage III tumors, wide excision may
be necessary. Chemotherapy or
radiation therapy are not indicated as
initial treatment, especially if this is an
isolated primary lesion.
z

Question 99 of 100

What are the most common primary sites for carcinomas metastatic
to bone?

1- Lung, breast, prostate, kidney, and thyroid

2- Lung, cervical, prostate, breast, and kidney

3- Breast, lung, prostate, bladder, and adrenal

4- Prostate, lung, breast, bladder, and cervical


z
discussion

The lung, breast, prostate, kidney, and thyroid are the most
common primary sites metastatic to bone. Thyroid cancer
metastases are becoming less common when patients pursue
treatment. Other metastatic disease including metastases from
cervical, hepatocellular, adrenal, and bladder cancers is less
common. Metastatic disease most commonly involves the spine
and pelvis.
z

Question 100 of 100

Figures 100a through 100c are the select T1 and T2 axial and T2 sagittal MR images of a 26
years old woman who has had intermittent left thigh pain for 6 months that is exacerbated by
activity. Left femur radiographs are unremarkable. Initial treatment should include

1- observation

2- sclerotherapy

3- open biopsy

4- wide resection
z

DISCUSSION

MR imaging reveals a lesion within the vastus intermedius muscle that has signal
characteristics consistent with vascular structures and a “bag-of-worms” appearance
consistent with a vascular malformation. T1-weighted MR imaging shows fat
interspersed among the tissue consistent with a vascular malformation. Vascular
malformations often can be determined on imaging alone, particularly if the
radiographs reveal phleboliths. Biopsy is generally unnecessary and open biopsy can
be particularly difficult due to significant bleeding if performed without the tourniquet,
and difficulty identifying the lesion if a tourniquet is utilized. Wide resection generally is
unnecessary because these lesions do not have malignant potential. Excision of
vascular malformations can be curative, but many patients experience symptom relief
with percutaneous sclerotherapy. This patient should be referred for consideration of
this less morbid minimally invasive approach.

You might also like